Você está na página 1de 105

Interbits – SuperPro ® Web

1. (Enem PPL 2020) Tanto a conservação de materiais biológicos como o resfriamento de


certos fotodetectores exigem baixas temperaturas que não são facilmente atingidas por
refrigeradores. Uma prática comum para atingi-las é o uso de nitrogênio líquido, obtido pela
expansão adiabática do gás N2 , contido em um recipiente acoplado a um êmbolo, que resulta
no resfriamento em temperaturas que chegam até seu ponto de liquefação em −196 C. A
figura exibe o esboço de curvas de pressão em função do volume ocupado por uma quantidade
de gás para os processos isotérmico e adiabático. As diferenças entre esses processos podem
ser identificadas com base na primeira lei da termodinâmica, que associa a variação de energia
interna à diferença entre o calor trocado com o meio exterior e o trabalho realizado no
processo.

A expansão adiabática viabiliza o resfriamento do N2 porque


a) a entrada de calor que ocorre na expansão por causa do trabalho contribui para a diminuição
da temperatura.
b) a saída de calor que ocorre na expansão por causa do trabalho contribui para a diminuição
da temperatura.
c) a variaçăo da energia interna é nula e o trabalho é associado diretamente ao fluxo de calor, que diminui
a temperatura do sistema.
d) a variação da energia interna é nula e o trabalho é associado diretamente à entrada de frio,
que diminui a temperatura do sistema.
e) o trabalho é associado diretamente à variação de energia interna e não há troca de calor
entre o gás e o ambiente.

2. (Unicamp 2020) O CO2 dissolvido em bebidas carbonatadas, como refrigerantes e


cervejas, é o responsável pela formação da espuma nessas bebidas e pelo aumento da
pressão interna das garrafas, tornando-a superior à pressão atmosférica. O volume de gás no
“pescoço” de uma garrafa com uma bebida carbonatada a 7 C é igual a 24 ml, e a pressão
no interior da garrafa é de 2,8  105 Pa. Trate o gás do “pescoço” da garrafa como um gás
perfeito. Considere que a constante universal dos gases é de aproximadamente 8 J mol  K e
que as temperaturas nas escalas Kelvin e Celsius relacionam-se da forma T(K) = 0(C) + 273.
O número de moles de gás no “pescoço” da garrafa é igual a
a) 1,2  105.
b) 3,0  103.
c) 1,2  10 −1.
d) 3,0  10−3.

3. (Fuvest 2023) O diamante, uma estrutura cristalina de carbono, é o material mais duro

Página 1 de 105
Interbits – SuperPro ® Web

encontrado na natureza. Mastigar diamantes provavelmente quebraria os dentes e a sua


ingestão causaria desconforto. Entretanto, há uma forma extravagante de ingerir um diamante,
que é utilizá-lo para produzir água com gás, como no seguinte procedimento:

1. O diamante triturado foi inserido dentro de um tubo de vidro;


2. Foi injetado oxigênio gasoso no tubo de vidro, que foi aquecido com um maçarico;
3. O gás produzido foi recolhido dentro de um tubo de ensaio imerso em nitrogênio líquido;
4. Todo o sólido formado dentro do tubo de ensaio foi transferido para um cilindro de gás de
volume interno de 1 L com a válvula fechada. Todo o CO 2 sublimou dentro desse cilindro;
5. O cilindro de gás foi então conectado a uma garrafa de água, que foi gaseificada com a
abertura da válvula do cilindro.

a) Com base no esquema, a temperatura de fusão do N2 é maior, menor ou igual a temperatura


de fusão do CO2?
b) Se um experimento exatamente igual for realizado trocando o volume de diamante utilizado
pelo mesmo volume de grafite, a quantidade de CO2 formado ao fim do processo será
menor, igual ou maior? Explique.
c) Partindo-se de 3 g de diamante, qual a massa de gelo seco obtida? Qual será a pressão
gerada pelo CO2(g) dentro do cilindro de gás a 27 °C? Considere que o rendimento do
experimento é de 100%.

Note e adote:
Densidade (g/cm3): Grafite = 2,0; Diamante = 3,5.
Massas molares (g/mol): C = 12; O = 16.
Constante universal dos gases ideais: R = 0,082 atm  L mol−1  K −1

4. (Unicamp 2023) Num estudo científico publicado em 2022 no Chemical Science, os autores
aqueceram amostras de casca seca de banana, utilizando, num curto período de tempo, uma
lâmpada de alta intensidade luminosa. Essa técnica de aquecimento denomina-se pirólise por
luz. Essa irradiação promoveu a formação de um tipo de carvão e produtos gasosos. Em um
experimento, a quantificação dos gases produzidos encontra-se na figura abaixo.

Página 2 de 105
Interbits – SuperPro ® Web

A partir dessas informações, pode-se inferir que um possível interesse dos autores era obter
combustíveis a partir de biomassa.
Sendo assim, pode-se dizer que o objetivo foi alcançado para hidrocarbonetos

Dados: C = 12; H = 1; O = 16.


a) leves, mas a maior quantidade em mol de gás foi a de hidrogênio.
b) leves, mas a maior quantidade em mol de gás foi a de monóxido de carbono.
c) pesados, mas a maior quantidade em mol de gás foi a de hidrogênio.
d) pesados, mas a maior quantidade em mol de gás foi a de monóxido de carbono.

5. (Ime 2023) Um combustível formado por uma mistura equimolar de n-propano e 2-metil-
propano alimenta a fornalha de uma usina termelétrica, na qual ocorre sua combustão total na
presença de ar. Um sensor posicionado na chaminé dessa fornalha detecta a presença de 3%
em mol de oxigênio nos gases de exaustão.

Calcule a razão ar/combustível, em proporção mássica, para uma alimentação de 1000 mol/s
desse combustível, com a fornalha operando sob essa condição.

Dados:
Composição do ar atmosférico: 79% de N2 e 21% de O2.
C = 12; H = 1; N = 14; O = 16.

6. (Ime 2023) Dois reatores A e B, com volumes invariáveis de 20 litros cada um, são
aquecidos até atingir a temperatura de 819 °C. Cada um dos reatores possui uma válvula de
segurança: a do reator A se abre automaticamente quando são produzidas em seu interior
pressões iguais ou superiores a 1,5 atm, enquanto que a do reator B se abre automaticamente
quando são produzidas em seu interior pressões iguais ou superiores a 3,5 atm.

No reator A foi armazenada hidrazina líquida (N2H4), que se decompôs inteiramente em 0,163
mol de gás hidrogênio e 0,082 mol de gás nitrogênio a 819 °C.
No reator B encontram-se em equilíbrio, amônia, 1,03  10 −2 mol L de N2 e 1,62  10 −2 mol L de
H2, a 819 °C, com um valor de KP (constante de equilíbrio em termos de pressão parcial) igual
a 0,25.

Dados:
- massas atômicas: H = 1 u; N = 14 u;

Página 3 de 105
Interbits – SuperPro ® Web

- R = 0,082atm  L (mol  K);e


- os gases se comportam idealmente.

Se aumentarmos em 10 °C a temperatura do reator A, podemos afirmar que:


a) a válvula do reator A se abre e a do reator B permanece aberta.
b) a válvula do reator A se abre e a do reator B permanece fechada.
c) a válvula do reator A permanece fechada e a do reator B permanece aberta.
d) as válvulas de ambos os reatores permanecem fechadas.
e) as válvulas de ambos os reatores permanecem abertas.

7. (Enem PPL 2022) Os airbags de segurança dos automóveis são acionados com o impacto,
que envia um sinal elétrico para o dispositivo e inicia a reação explosiva do trinitreto de sódio
(NaN3), produzindo sódio metálico e nitrogênio molecular, conforme a equação:
2NaN3(s) ⎯⎯→ 2Na(s) + 3N2(g)

O gás produzido tem função de inflar o airbag. Esse tipo de dispositivo contém,
aproximadamente, 100 g de NaN3.

Considere:
PV = nRT; P = 1 atm; T = 25 °C;
L  atm
R = 0,0821 ; 0 C = 273K
K  mol
g g g
Massas molares: NaN3 = 65 ; N2 = 28 ; Na = 23
mol mol mol

Nesse dispositivo, o volume de gás produzido, em litro, é


a) 4,7.
b) 9,4.
c) 18,8.
d) 56,5.
e) 113,0.

8. (Ita 2022) Em um reator químico vazio, mantido a altas temperaturas, injeta-se uma mistura
gasosa, com massa molar aparente igual a 29 g  mol−1, constituída de amônia e oxigênio
puros. Os gases reagem entre si formando óxido nítrico e água no estado gasoso. Ao final do
processo, toda a amônia é consumida e são formados 20 mol de óxido nítrico. A respeito deste
processo, pede-se:

a) A equação química balanceada.


b) Os valores numéricos das frações molares de amônia e de oxigênio no início da reação.
c) O valor numérico da porcentagem de reagente em excesso.
d) Os valores numéricos das quantidades (em mols) das espécies químicas no final da reação.

Dados: N = 14; H = 1; O = 16.

9. (Ita 2022) Na reação conhecida como “cão que late”, uma mistura de óxido nitroso e
dissulfeto de carbono entra em combustão, gerando um clarão azulado e um som parecido
com “woof” ou “uulsh”. Considerando uma combustão completa e que todo o enxofre gerado se
encontra na forma de sólido S8:

a) escreva a equação química balanceada dessa reação.


b) determine o valor numérico do volume de gás gerado (em litros) para cada 304 g de
dissulfeto de carbono que reagiu de forma estequiométrica. Considere a pressão igual a 10 5
Pa e a temperatura de 300 K.
c) calcule o valor numérico da massa de enxofre sólido (em g) gerado considerando a mesma
quantidade de dissulfeto de carbono do item (b).

Página 4 de 105
Interbits – SuperPro ® Web

10. (Upe-ssa 2 2022) Em um episódio da série “Profissão Perigo”, o agente secreto MacGyver,
protagonista do seriado, está em fuga. Quando entra em um depósito de produtos químicos,
percebe que, em um dos lados, há um tambor, cujo rótulo indica conter hidróxido de amônio
(NH4 OH); de outro lado, observa que há um tambor contendo ácido muriático (solução aquosa
de HC ). Ele abre os dois recipientes e segue sua trajetória de fuga. Logo, forma-se uma
espessa nuvem branca, que impede a passagem de seus perseguidores.
A nuvem branca é devida à formação de cloreto de amônio (NH4 C ), a partir da reação entre
os gases amônia (NH3) e HC , liberados de cada recipiente, quando abertos.

Dados: Massas molares (g/mol): H = 1; N = 14; C = 35,5. Despreze a diferença de tempo


entre a abertura dos recipientes.

NH3(g) HC (g) NH4 C (s)


Entalpia padrão de formação (kJ/mol) – 46,1 – 92,3 – 314,6

A partir da descrição apresentada, assinale a alternativa CORRETA.


a) A transformação acontece com absorção de calor do meio externo.
b) A transformação é um exemplo de equilíbrio químico homogêneo.
c) A nuvem branca começa a se formar em um ponto equidistante aos dois recipientes.
d) O cloreto de amônio é um sal, que, quando dissolvido em água, forma uma solução alcalina.
e) A nuvem branca começa a se formar em um ponto mais próximo do recipiente que contém o
ácido muriático.

11. (Ita 2022) Um químico carregou um reator com 20 atm de uma mistura gasosa, constituída
de uma substância A e de um componente inerte I, em uma proporção molar de A:I igual a 4:1.
A temperatura do reator foi mantida constante e a pressão total foi monitorada, o que permitiu
determinar a velocidade da reação em função do tempo, de acordo com os dados da tabela.

t(min) 0,89 2,08 3,75 6,25 10,42 18,75


P (atm) 21 22 23 24 25 26
v(atm  min−1) 1,96 1,44 1,00 0,64 0,36 0,16

Com base nesses dados e sabendo que a estequiometria da reação é 2A(g) ⎯⎯→ 3B(g),
pede-se:

a) O valor numérico da ordem da reação.


b) O valor numérico da constante de velocidade com sua unidade de medida.
c) A composição no interior do reator no tempo 10,42 minutos em termos das pressões parciais
(em atm) de cada componente.
d) O valor numérico do tempo de meia vida da reação.

12. (Fuvest 2022) Nas vias públicas, é possível encontrar totens conforme os representados
na figura. Em um deles há a inscrição “COMGAS”, indicando que ali passa um gasoduto da
empresa distribuidora de gás; no outro está escrito “PROT CATÓDICA”, o que indica que há
uma proteção catódica no gasoduto.

Página 5 de 105
Interbits – SuperPro ® Web

Sabendo que o gasoduto é constituído de aço, responda:

a) No sistema do gasoduto apresentado, o gasoduto é o ânodo ou o cátodo da célula


eletroquímica? Justifique a sua resposta.

b) Considere os seguintes potenciais de redução dos metais (ε) :

Semirreação ε (V)
2+ − 0
Cu + 2 e ⎯⎯→ Cu + 0,34

Sn2+ + 2 e− ⎯⎯→ Sn0 − 0,14

Fe2+ + 2 e− ⎯⎯→ Fe0 − 0,44

Zn2+ + 2 e− ⎯⎯→ Zn0 − 0,76

Qual(is) desses metais pode(m) ser utilizado(s) para a proteção catódica do gasoduto?
Esse(s) metal(is) será(ão) o polo positivo ou negativo da célula eletroquímica?

c) Considere que a composição molar do gás transportado pelo gasoduto Brasil-Bolívia é de,
aproximadamente, 93,5% metano, 4,1% etano e 2,4% N2. Calcule a massa, em kg, de etano
existente em uma seção de 1 m3 do duto sabendo que a pressão no interior do duto é de
100 atm à temperatura de 27 °C. Apresente os cálculos.

Note e adote:
Constante universal dos gases ideais: R = 8,2  10−5 m3 atm K −1 mol−1.
T(K) = T(°C) + 273
Massas molares (g/mol): H = 1; C = 12.

13. (Ime 2022) Bicarbonato de sódio reage estequiometricamente, em processo isotérmico a


300 K, com 50 mL de uma solução aquosa de um ácido monoprótico forte, em um recipiente
rígido e fechado que, quando vazio, apresenta um volume útil de 74,9 mL, conforme o esboço
abaixo:

Página 6 de 105
Interbits – SuperPro ® Web

O cristal piezoelétrico tem espessura (t) de 2 mm e suscetibilidade voltaica (Φ) de


0,050 V  (m  Pa)−1 e, quando a reação atinge o equilíbrio, fornece um potencial elétrico (U) de
1,0V. Considere a solubilidade molar de gases na água desprezível e a constante universal dos
gases R = 8,3 J  (mol  K)−1. Se o volume reacional é constante e igual ao volume da solução
ácida inicial, a concentração molar inicial da solução do ácido monoprótico, em mol  L−1, é:
a) 6
b) 2
c) 0,6
d) 0,002
e) 0,006

14. (Unicamp 2022) O sistema PSA (Pressure Swing Adsorption) é uma alternativa à produção
de oxigênio para hospitais ou indústrias distantes de fornecedores. Esse sistema é também
uma opção para os casos em que a produção por destilação criogênica não supre a demanda.
O funcionamento do PSA se baseia na adsorção seletiva do oxigênio ou do nitrogênio. O ar
pressurizado é forçado a passar por colunas contendo substâncias sólidas. Tais substâncias
adsorvem seletivamente ou o oxigênio ou o nitrogênio. Quando o sólido fica saturado,
promove-se uma despressurização do sistema e então o gás se dessorve do sólido. Dessa
forma, os dois principais gases do ar são separados um do outro. A figura A esquematiza o
sistema de funcionamento do PSA e os gráficos 1 e 2 representam isotermas de adsorção
(estudo do equilíbrio a uma temperatura constante) de N2 e O2 para dois adsorventes distintos.
O gráfico 1 diz respeito a um complexo do metal cobalto denominado CoBTTri, enquanto o
gráfico 2 diz respeito a um zeólito representado pela fórmula NaX.

Página 7 de 105
Interbits – SuperPro ® Web

Considerando as figuras, é correto afirmar que a separação a que se refere o texto seria
possível utilizando-se
a) apenas o material NaX. O oxigênio seria obtido na pressurização do sistema.
b) qualquer um dos dois materiais. O oxigênio seria obtido na despressurização do sistema
apenas no caso do CoBTTri.
c) apenas o material CoBTTri. O oxigênio seria obtido na pressurização do sistema.
d) qualquer um dos dois materiais. O oxigênio seria obtido na despressurização do sistema
apenas no caso do NaX.

15. (Espcex (Aman) 2021) Gases apresentam um fenômeno chamado de difusão de gases.
“Difusão gasosa é o movimento espontâneo das partículas de um gás de se espalharem
uniformemente em meio das partículas de um outro gás ou de atravessarem uma parede
porosa.
Quando gases diferentes (A e B) estão a uma mesma temperatura, para uma quantidade de
matéria igual a 1 mol de gás A e 1 mol de gás B, tem-se que a massa do gás A (ma ) será
igual à massa molar do gás A (Ma ), e a massa do gás B (mb ) será igual à massa molar do
gás B (Mb )."

FONSECA, Martha Reis Marques da, Química Geral, São Paulo, Ed FTD, 2007, Pág. 110

Considere que, em determinadas condições de temperatura e pressão, a velocidade de difusão


de 1 mol do gás hidrogênio (H2 ) seja de 28 km min. Nestas mesmas condições a velocidade
(em km h) de 1 mol do gás metano (CH4 ) é de

Dados: 2 = 1,4; C = 12; H = 1.

a) 600 km h
b) 729 km h
c) 1211km h
d) 422 km h
e) 785 km h

16. (Fgv 2021) Em um procedimento de destilação cujo objetivo era recolher o produto em
temperaturas baixas, o recipiente de coleta, submetido inicialmente a 1 atm e à temperatura de
27°C, foi mergulhado em um frasco construído com isolamento térmico contendo nitrogênio
líquido à temperatura de –196°C, conforme representado na figura.

Página 8 de 105
Interbits – SuperPro ® Web

Após alcançado o equilíbrio térmico no interior do recipiente de coleta, o valor percentual da


pressão em seu interior, em relação à pressão inicial, será, aproximadamente,
a) 14%.
b) 26%.
c) 38%.
d) 50%.
e) 77%.

17. (Unicamp 2021) A pressão insuficiente, em excesso ou desigual entre os pneus coloca em
risco a segurança na condução e afeta o rendimento do veículo. Pensando nisso, numa manhã
fria (10 C), um motorista efetuou corretamente a calibração dos pneus do seu carro para 29 e
31 psi, seguindo a tabela de calibragem dos pneus no manual do fabricante, como indica a
figura a seguir. Ao meio-dia, chegou ao seu destino e, após um período de descanso, carregou
o carro com lotação máxima.

Considerando que a temperatura ambiente naquele momento era de 30 C, o motorista


certamente precisaria

Página 9 de 105
Interbits – SuperPro ® Web

Dados: T K = 273 + t C; desconsiderar a variação no volume dos pneus; o sensor de pressão
não indica variações menores que 1 psi.
a) encher os pneus dianteiros e traseiros.
b) encher os pneus dianteiros e esvaziar os traseiros.
c) encher apenas os pneus traseiros.
d) encher apenas os pneus dianteiros.

18. (Unicamp 2021) Um dos grandes desafios para a consolidação de uso do hidrogênio como
combustível é seu
armazenamento seguro e em grande quantidade. O hidrogênio pode ser armazenado puro,
como gás ou líquido. Atualmente, parece mais adequado armazenar o hidrogênio na forma de
hidretos metálicos ou adsorvido em materiais porosos nanoestruturados. Para que o
armazenamento seja considerado eficiente, o material deve apresentar capacidade de
armazenamento máxima em pressão constante e boa reversibilidade; ou seja, o
armazenamento (adsorção) e a liberação (dessorção) devem ocorrer em condições similares.
Essas características do armazenamento podem ser observadas em um gráfico denominado
“isoterma de adsorção”, que é uma curva de composição de hidrogênio no material (C, kg de
H2 kg de material) em função da pressão.

a) A figura mostra a isoterma de três materiais que poderiam ser empregados para armazenar
H2 . Qual curva (A, B ou C) representa o melhor material para se armazenar o hidrogênio?
Justifique sua escolha.

b) Um carro com motor a combustão interna consome 24 kg de gasolina (d = 700 kg m−3 ) ou


8 kg de hidrogênio para percorrer uma distância de 400 km, adsorvido em um material
intermetálico do tipo Mg2Ni. Considerando que a massa e o volume de um carro médio são
aproximadamente de 6 m3 e 1.000 kg, respectivamente, uma possível desvantagem desta
tecnologia alternativa estaria relacionada à massa ou ao volume relativamente ocupado
pelo Mg2Ni? Justifique.

Dados do Mg2Ni : capacidade de armazenamento de H2 = 3,6 kg de H2 por 100 kg de


Mg2Ni; densidade = 3.400 kg m−3 .

19. (Ime 2021) O modelo dos gases ideais, ou perfeitos, descreve bem o comportamento para
a maioria dos casos, no entanto, foi necessário desenvolver modelos mais precisos dentre os
quais se destaca a equação de Van der Waals. Deduza a equação de Van der Waals,
assumindo que o volume da partícula/molécula não seja desprezível e existam interações entre

Página 10 de 105
Interbits – SuperPro ® Web

as partículas/moléculas. Considere o seguinte:

- V é o volume do recipiente do gás;


- B é o volume total ocupado pelas moléculas do gás;
- As forças de atração são praticamente nulas no seio da mistura do gás: e
- Próximo às paredes do recipiente, as moléculas são atraídas ao centro com uma força
proporcional ao quadrado da concentração do gás, o que reduz a intensidade dos impactos
nas paredes do recipiente.

20. (Ita 2021) Considere a combustão de um determinado alceno com uma quantidade
definida de ar em excesso. Considere, ainda, que o ar é composto apenas por nitrogênio e
oxigênio numa proporção molar de 3,76 e que o nitrogênio não sofre oxidação durante a
combustão. Para esta reação, determine a porcentagem do(s) gás(es) em cada uma das
situações descritas abaixo.

a) Na combustão incompleta do alceno com ar em excesso, além dos produtos esperados


numa combustão completa, há a formação de monóxido de carbono na proporção molar de
1 para 8 em relação ao dióxido de carbono. Além disso, a quantidade em mol de O2 que
sobrou após a reação é igual à de carbono no início da reação. Com base nessas
informações, determine a composição percentual aproximada dos gases resultantes da
reação, considerando a remoção prévia da água.
b) Determine o percentual de ar em excesso na reação de combustão completa do alceno.

21. (Unicamp 2021) “Hospital Municipal de Juruti (PA) recebe mais de 70 cilindros de oxigênio
para tratar pacientes com Covid-19” (site G1, 01/06/2020). A oxigenoterapia é indicada para
todos os pacientes graves, inicialmente variando de 5 a 10 L de O2 min. Para uma vazão
constante e máxima na faixa considerada, o cilindro de cada paciente deverá,
necessariamente, ser trocado após aproximadamente

Dados: volume interno do cilindro = 50 L; volume aproximado do gás a 1 atm de pressão em


cada cilindro = 10 m3 ; pressão inicial no cilindro = ~ 200 atm.

a) 17 horas de uso, sendo o volume de gás restante no cilindro igual a 50 L e a pressão 1 atm.
b) 33 horas de uso, sendo o volume de gás restante no cilindro igual a 50 L e a pressão
0 atm.
c) 33 horas de uso, sendo o volume de gás restante no cilindro igual a 0 L e a pressão 0 atm.
d) 17 horas de uso, sendo o volume de gás restante no cilindro igual a 0 L e a pressão 1 atm.

22. (Unicamp 2021) O gás natural (GN) é uma fonte de energia eficiente e limpa,
considerando-se uma queima completa desse gás na sua forma purificada. No entanto, o
metano, na origem, vem misturado com muitas outras substâncias que precisam ser retiradas
no processo de purificação, pois, na queima, teriam baixa ou nenhuma eficiência energética, ou
então gerariam produtos com características indesejáveis. A tabela abaixo mostra a
composição aproximada (V V %) de algumas fontes de gás natural, o que pode ilustrar as
afirmações anteriores.

Eficiência Energética Fontes


Componente 3
Relativa (MJ m ) Laeq Uch Uthmaniyah
CH4 37 69 27,3 55,5
C2H6 65 3 0,7 18
C3H8 92 0,9 0,3 9,8
C4H10 120 0,5 0,3 4,5
C5 + 147 0,5 - 1,6
N2 0 1,5 25,2 0,2

Página 11 de 105
Interbits – SuperPro ® Web

H2S 22 15,3 - 1,5


CO2 0 9,3 46,2 8,9

Considere a queima de gases naturais (GN) na composição em que se apresentam nas fontes,
em condições idênticas de temperatura e pressão e considerando tais gases como ideais.

a) Do ponto de vista energético, qual seria a melhor e a pior fonte, por volume de gás
queimado? Justifique sua resposta.
MELHOR FONTE:
PIOR FONTE:
Justificativa:

b) Do ponto de vista ambiental, qual seria a melhor e a pior fonte, por volume de gás
queimado? Justifique sua resposta.
MELHOR FONTE:
PIOR FONTE:
Justificativa:

23. (Ita 2021) Um reator químico, projetado com uma válvula de alívio de pressão que é
acionada a 8,5 atm, contém uma mistura gasosa composta por quantidades iguais de um
reagente (A) e de uma substância inerte (B), a 10 C e 2 atm. Ao elevar rapidamente a
temperatura do reator para 293 C, o reagente A começa a se decompor de acordo com a
seguinte equação estequiométrica genérica:

2A (g) → 3C(g) + 4D(g) + E(g)

Sabendo que a velocidade de consumo de A nessa temperatura é dada por v A = −0,25  (PA )0
(em atm  h−1 ), onde PA corresponde à pressão parcial da substância A, responda:

a) Após quanto tempo de reação a válvula de alívio é acionada?


b) Quais as pressões parciais de cada espécie (A, B, C, D e E) presente no reator no momento
do acionamento da válvula de alívio?
c) Assumindo 100% de rendimento da reação, qual a quantidade máxima de mistura gasosa
que pode ser adicionada ao reator sem que a válvula de alívio seja acionada?
Dado: R = 8,21 10−2 atm  L  K −1  mol−1.

24. (Fgv 2020) Um experimento para a identificação de um metal M foi realizado de acordo
com a montagem instrumental da figura.

Página 12 de 105
Interbits – SuperPro ® Web

A solução de HC foi adicionada até que toda amostra do metal M, de massa 2,38 g, reagisse
completamente, formando gás hidrogênio (H2 ), cujo volume coletado, a 27 C e 1,00 atm, foi
de 480 mL.

Considerando que a constante geral dos gases seja R = 0,08 atm  L  mol−1  K −1, o metal
empregado nesse experimento foi o

Dados: Fe = 55,8; Ni = 58,7; Zn = 65,4; Sn = 119; Pb = 207.


a) zinco.
b) estanho.
c) chumbo.
d) níquel.
e) ferro.

25. (Espcex (Aman) 2020) Um analista químico realizou um experimento em que utilizou
200 mL de uma solução de concentração 2 mol L de ácido clorídrico (HC ) para reagir com
uma certa massa de bicarbonato de sódio (também denominado de hidrogenocarbonato de
sódio). Notou que nem todo o ácido reagiu com essa massa de bicarbonato de sódio, restando
um excesso de ácido. Ao final do experimento, ele obteve um volume de 6,15 L de gás
carbônico, medidos a 27 C e 1 atm. Esse gás carbônico é oriundo da decomposição do ácido
carbônico produzido na reação.

Dados: R = 0,082 atm  L  mol−1  K −1


T(Kelvin) = t(Celsius) + 273

Esse analista fez as seguintes afirmativas:

I. A equação química balanceada que descreve corretamente a reação citada é:


Na2CO3 + 2 HC → H2CO3 + 2 NaC
II. Para a formação de 6,15 L de CO2 , foram consumidos 21 g de bicarbonato de sódio.
III. É uma reação de oxidorredução e o ácido clorídrico é o agente oxidante.
IV. Se todo esse ácido clorídrico fosse consumido numa reação completa com bicarbonato de
sódio suficiente, o volume de gás carbônico produzido seria de 9,84 L.

Dado: Na = 23; H = 1; C = 12; O = 16.

Assinale a alternativa que apresenta todas as afirmativas corretas, dentre as listadas acima.
a) I, II e III.
b) II e III.
c) III e IV.
d) II e IV.
e) II, III e IV.

26. (Ita 2020) Um reator com 200 L de capacidade, possui uma mistura de dióxido de
nitrogênio e monóxido de carbono a 400 K, cujo comportamento pode ser considerado ideal.
Os gases reagem entre si para formar dióxido de carbono e monóxido de nitrogênio. A pressão
total no reator é igual a 32,8 atm e, no início da reação, a pressão parcial do monóxido de
carbono é três vezes maior que a do dióxido de nitrogênio. As massas iniciais de dióxido de
nitrogênio e de monóxido de carbono são, respectivamente,
a) 1,5 kg e 4,2 kg.
b) 1,5 kg e 4,5 kg.
c) 1,5 kg e 6,6 kg.
d) 2,3 kg e 4,2 kg.

Página 13 de 105
Interbits – SuperPro ® Web

e) 2,3 kg e 6,6 kg.

27. (Fuvest 2019) Um grão de milho de pipoca, visto a olho nu, apresenta duas regiões
distintas, representadas por A e B na figura. Em A, ocorre o tecido acumulador de amido,
usado, pela planta, para nutrir o embrião. Em B, os tecidos vegetais possuem maior teor de
água. Ao ser aquecida, parte da água transforma-se em vapor, aumentando a pressão interna
do grão. Quando a temperatura atinge 177 C, a pressão se torna suficiente para romper o
grão, que vira uma pipoca.

Um estudo feito por um grupo de pesquisadores determinou que o interior do grão tem 4,5 mg
de água da qual, no momento imediatamente anterior ao seu rompimento, apenas 9% está na
fase vapor, atuando como um gás ideal e ocupando 0,1 mL. Dessa forma, foi possível calcular
a pressão Pfinal no momento imediatamente anterior ao rompimento do grão.

A associação correta entre região do milho e Pfinal é dada por:

Note e adote:
- Constante universal dos gases: R = 0,082 L  atm (K  mol);
- K = C + 273;
- Massas molares (g mol) : H = 1; O = 16.
a) A = endosperma e Pfinal = 8,3 atm.
b) B = endosperma e Pfinal = 5,9 atm.
c) A = xilema e Pfinal = 22,1 atm.
d) B = xilema e Pfinal = 5,9 atm.
e) B = endosperma e Pfinal = 92,0 atm.

28. (Ita 2019) Uma amostra de gás contém 80% de metano, 10% de etano, 5% de propano
e 5% de nitrogênio, em volume. Considerando que todos os átomos de carbono na amostra de
gás são convertidos em butadieno com 100% de rendimento, assinale a opção que apresenta
a massa de butadieno obtido a partir de 100 g do gás.
a) 50 g
b) 60 g
c) 70 g
d) 80 g
e) 90 g

29. (Unicamp 2019) sEpisódios recentes de erupções vulcânicas têm trazido consequências
trágicas para a sociedade e para o meio ambiente. Ativo desde 1983, o Vulcão Kilauea
apresentou, em 2018, a sua maior erupção já registrada. Quase ao mesmo tempo, foi a vez do
Vulcão Fuego da Guatemala mostrar sua força. No Kilauea não houve explosões, ao contrário
do que ocorreu no Fuego. Os especialistas afirmam que a ocorrência de uma erupção
explosiva depende da concentração e do tipo de gases dissolvidos no magma, como SO2 , HF
e HC , além de vapor de água e CO2 aprisionados. A figura a seguir dá informações sobre a
relação entre quantidades (em mol) de SO2 , HF e HC no magma de três vulcões distintos.

Página 14 de 105
Interbits – SuperPro ® Web

De acordo com a figura, em relação às quantidades de gases dissolvidos no magma, é correto


afirmar que as concentrações de SO2 são maiores que as de HF e de HC
a) nos três vulcões e, neles, HF e HC são aproximadamente iguais.
b) em apenas dois vulcões e, neles, HF e HC são aproximadamente iguais.
c) nos três vulcões, mas em apenas dois deles HF e HC são aproximadamente iguais.
d) em apenas dois vulcões, mas nos três vulcões HF e HC são aproximadamente iguais.

30. (Fgv 2018) Uma substância gasosa X, massa molar 32 g mol, apresenta densidade igual
a 2,0 g L a uma certa condição de temperatura e pressão. Nessas mesmas condições de
temperatura e pressão, uma outra substância gasosa Y tem densidade igual a 3,0 g L.

A massa molar da substância Y em g mol é


Dado: P  V = n  R  T
a) 72.
b) 48.
c) 36.
d) 24.
e) 10.

31. (Fuvest 2018) Em navios porta-aviões, é comum o uso de catapultas para lançar os aviões
das curtas pistas de decolagem. Um dos possíveis mecanismos de funcionamento dessas
catapultas utiliza vapor de água aquecido a 500 K para pressurizar um pistão cilíndrico de
60 cm de diâmetro e 3 m de comprimento, cujo êmbolo é ligado à aeronave.

Após a pressão do pistão atingir o valor necessário, o êmbolo é solto de sua posição inicial e o
gás expande rapidamente até sua pressão se igualar à pressão atmosférica (1 atm). Nesse
processo, o êmbolo é empurrado, e o comprimento do cilindro é expandido para 90 m,
impulsionando a aeronave a ele acoplada.

Esse processo dura menos de 2 segundos, permitindo que a temperatura seja considerada
constante durante a expansão.

a) Calcule qual é a pressão inicial do vapor de água utilizado nesse lançamento.

Página 15 de 105
Interbits – SuperPro ® Web

b) Caso o vapor de água fosse substituído por igual massa de nitrogênio, nas mesmas
condições, o lançamento seria bem sucedido? Justifique.

Note e adote:
Constante universal dos gases: R = 8  10−5 atm m3mol−1K −1; π = 3;
Massas molares:
H2O.....18 g mol
N2 .....28 g mol

32. (Ime 2018) Um sistema fechado contendo um gás ideal no estado 1 sofre as
transformações α e β, conforme indicado na figura abaixo.

Sabendo que a transformação α é isotérmica e β isobárica, indique o gráfico que representa


os estados do sistema.

a)

b)

c)

Página 16 de 105
Interbits – SuperPro ® Web

d)

e)

33. (Ita 2018) Um recipiente de 240 L de capacidade contém uma mistura dos gases ideais
hidrogênio e dióxido de carbono, a 27 C. Sabendo que a pressão parcial do dióxido de
carbono é três vezes menor que a pressão parcial do hidrogênio e que a pressão total da
mistura gasosa é de 0,82 atm, assinale a alternativa que apresenta, respectivamente, as
massas de hidrogênio e de dióxido de carbono contidas no recipiente.
a) 2 g e 44 g
b) 6 g e 44 g
c) 8 g e 88 g
d) 12 g e 88 g
e) 16 g e 44 g

34. (Ita 2018) Uma dada reação (I), cujo calor liberado é desconhecido, é conduzida em um
reator que utiliza um gás mantido a volume constante (V) como banho térmico. Outras duas
reações (II e III) conduzidas em condições similares apresentam calor liberado a volume
constante (Q V ) conforme apresentado na tabela abaixo:

Reação Equação Q V (kJ mol−1)


I A+ 1 B →D ?
2
II A +B → C 400
III D+ 1 B →C 300
2

Considere as seguintes informações sobre o gás do banho térmico, que tem comportamento
não ideal e obedece à equação:

 n2a 
 P + 2  (V − nb) = nRT,
 V 

em que: a = 62,5 L2 atm mol−1; b = 0,4 L mol−1; n = 0,4 mol; V = 10 L; capacidade calorífica
molar a volume constante (CV,m ) = 83,33 J K −1 mol−1; temperatura inicial (Ti ) = 300 K.

Página 17 de 105
Interbits – SuperPro ® Web

a) Sabendo que 0,1 mol de A são utilizados na reação I, calcule o Q V liberado nessa reação.
b) Determine a temperatura final do banho térmico.
c) Determine a pressão inicial e a pressão final do banho térmico.

35. (Mackenzie 2018) Certa massa fixa de um gás ideal, sob temperatura de 30 C e pressão
de 2 atm, foi submetida a uma transformação isocórica, em que sua temperatura foi
aumentada em 150 unidades. Dessa forma, é correto afirmar que, durante a transformação,
a) além do volume, a pressão manteve-se constante.
b) apenas o volume permaneceu constante, e no final, a pressão exercida por essa massa
gasosa, foi aumentada para aproximadamente 12 atm.
c) apenas o volume permaneceu constante, e no final, a pressão exercida por essa massa
gasosa, foi aumentada para aproximadamente 3 atm.
d) apenas o volume permaneceu constante, e no final, a pressão exercida por essa massa
gasosa, foi diminuída para aproximadamente 1 atm.
e) apenas o volume permaneceu constante, e no final, a pressão exercida por essa massa
gasosa, foi diminuída para aproximadamente 0,33 atm.

36. (Ime 2018) Em um vaso fechado, ocorreu a reação de 13,1 gramas de Xe(g) com excesso
de F2(g) cuja pressão parcial é de 2,4 atm e a pressão total de 6 atm. Tal reação formou
exclusivamente o composto apolar A, que possui 14 pares de elétrons não ligantes. Em
seguida, foram adicionados 19,5 g de platina na forma sólida, que reagiram exclusivamente
com o composto A para formar um produto X, recuperando o gás nobre.

Considerando comportamento de gás ideal e sabendo que as reações ocorreram à


temperatura de 400 C, determine:

a) a massa de flúor que não reagiu;


b) a estrutura de Lewis do composto A; e
c) a massa do produto X obtido.

37. (Ita 2018) Considere um recipiente de 320 L, ao qual são adicionados gases ideais nas
seguintes condições:

I. Hélio: 30.000 cm3 a 760 cmHg e 27 C


II. Monóxido de carbono: 250 L a 1.140 mmHg e −23 C
III. Monóxido de nitrogênio: 2 m3 a 0,273 atm e 0 C

Sabendo que a pressão total da mistura gasosa é de 4,5 atm, assinale a opção que apresenta
a pressão parcial do hélio na mistura gasosa.
a) 0,1 atm
b) 0,2 atm
c) 0,5 atm
d) 1,0 atm
e) 2,0 atm

38. (Unicamp 2017) Um teste caseiro para saber se um fermento químico ainda se apresenta
em condições de bom uso consiste em introduzir uma amostra sólida desse fermento em um
pouco de água e observar o que acontece. Se o fermento estiver bom, ocorre uma boa
efervescência; caso contrário, ele está ruim. Considere uma mistura sólida que contém os íons
dihidrogenofosfato, H2 PO 4− , e hidrogenocarbonato, HCO3− .

Página 18 de 105
Interbits – SuperPro ® Web

a) Considerando que o teste descrito anteriormente indica que a mistura sólida pode ser de um
fermento que está bom, escreva a equação química que justifica esse resultado.
b) Tendo em vista que a embalagem do produto informa que 18 g desse fermento químico
devem liberar, no mínimo, 1,45  10−3 m3 de gases a 298 K e 93.000 Pa, determine a
mínima massa de hidrogenocarbonato de sódio que o fabricante deve colocar em 18
gramas do produto.

Dado: R = 8,3 Pa m3 mol−1 K −1.

39. (Ita 2017) Após inalar ar na superfície, uma pessoa mergulha até uma profundidade de
200 m, em apneia, sem exalar. Desconsiderando as trocas gasosas que ocorrem nos alvéolos
pulmonares, calcule a pressão parcial do nitrogênio e do oxigênio do ar contido no pulmão do
mergulhador.

40. (Unicamp 2017) Bebidas gaseificadas apresentam o inconveniente de perderem a graça


depois de abertas. A pressão do CO2 no interior de uma garrafa de refrigerante, antes de ser
aberta, gira em torno de 3,5 atm, e é sabido que, depois de aberta, ele não apresenta as
mesmas características iniciais. Considere uma garrafa de refrigerante de 2 litros, sendo
aberta e fechada a cada 4 horas, retirando-se de seu interior 250 mL de refrigerante de cada
vez.

Nessas condições, pode-se afirmar corretamente que, dos gráficos a seguir, o que mais se
aproxima do comportamento da pressão dentro da garrafa, em função do tempo é o

a)

b)

c)

Página 19 de 105
Interbits – SuperPro ® Web

d)

41. (Fuvest 2017) Os pneus das aeronaves devem ser capazes de resistir a impactos muito
intensos no pouso e bruscas alterações de temperatura. Esses pneus são constituídos de uma
câmara de borracha reforçada, preenchida com o gás nitrogênio (N2 ) a uma pressão típica de
30 atm a 27 C. Para a confecção dessa câmara, utiliza-se borracha natural modificada, que
consiste principalmente do poli-isopreno, mostrado a seguir:

−CH2 − C(CH3 ) = CH − CH2 − CH2 − C(CH3 ) = CH − CH2 − CH2 − C(CH3 ) = CH − CH2 −

Em um avião, a temperatura dos pneus, recolhidos na fuselagem, era −13 C durante o voo.
Próximo ao pouso, a temperatura desses pneus passou a ser 27 C, mas seu volume interno
não variou.

a) Qual é a pressão interna de um dos pneus durante o voo? Mostre os cálculos.


b) Qual é o volume interno desse mesmo pneu, em litros, dado que foram utilizados 14 kg de
N2 para enchê-lo? Mostre os cálculos.
c) Escreva a fórmula estrutural do monômero do poli-isopreno.

Note e adote:
Massa molar do N2 = 28 g mol
Constante universal dos gases = 0,082 L  atm  K −1  mol−1
K = C + 273

42. (Ita 2017) Um frasco fechado contém dois gases cujo comportamento é considerado ideal:
hidrogênio molecular e monóxido de nitrogênio. Sabendo que a pressão parcial do monóxido
de nitrogênio é igual a 3 5 da pressão parcial do hidrogênio molecular, e que a massa total da
mistura é de 20 g, assinale a alternativa que fornece a porcentagem em massa do hidrogênio
molecular na mistura gasosa.
a) 4%
b) 6%
c) 8%
d) 10%
e) 12%

TEXTO PARA AS PRÓXIMAS 2 QUESTÕES:


Um motor pulso-jato é uma máquina térmica que pode ser representada por um ciclo
termodinâmico ideal de três etapas:

I. Aquecimento isocórico (combustão).


II. Expansão adiabática (liberação de gases).
III. Compressão isobárica (rejeição de calor a pressão atmosférica).

43. (Ita 2017) Considerando que essa máquina térmica opere com gases ideais, indique qual
dos diagramas pressão versus volume a seguir representa o seu ciclo termodinâmico.

Página 20 de 105
Interbits – SuperPro ® Web

a)

b)

c)

d)

e)

44. (Ita 2017) Sobre este motor pulso jato, considere verdadeiras as seguintes afirmações:

A. A temperatura de fusão do material que compõe a câmara de combustão é 1.500 K, e acima


de 1.200 K o material do motor começa a sofrer desgaste considerável pelos gases de
combustão;
B. O material do motor resiste a pressões de até 30 atm;

Página 21 de 105
Interbits – SuperPro ® Web

C. O motor opera, em cada ciclo termodinâmico, com 0,2 mol de uma mistura de gases com
comportamento ideal, iniciando o ciclo em pressão atmosférica e a temperatura de 300 K.

a) A partir destas informações e considerando que se deseja obter, de forma segura, o máximo
de trabalho por ciclo, quais devem ser a pressão e a temperatura no ponto de intersecção
entre os processos I e II do ciclo termodinâmico?

b) Na mistura de gases que opera em cada ciclo há uma fração de combustível, o qual tem a
reação de combustão dada por:

CH4(g) + 2 O2(g) → CO2(g) + 2 H2O(g) Qv = 45 kJ  g−1

em que Qv é o calor liberado a volume constante, por grama de metano. Considerando a


capacidade calorífica molar a volume constante da mistura de gases igual a
25 J  K −1  mol−1, qual é a massa de metano utilizada pelo ciclo projetado no item anterior?

45. (Mackenzie 2016) 11,2 g de sucata, contendo ferro, reagiram com quantidade suficiente de
ácido clorídrico em solução produzindo solução de cloreto de ferro II e gás hidrogênio. O gás
formado foi aprisionado em um balão com 1L de volume, exercendo uma pressão de
2,46 atm, sob temperatura de 27 C. Considerando-se que somente o ferro que reagiu seja
capaz de produzir o gás hidrogênio, é possível afirmar que o teor de ferro, na sucata, é de

Dados:
- massa molar (g mol−1 ) Fe = 56
- constante universal dos gases ideais (R) = 0,082 atm L  mol−1 K
a) 90%
b) 80%
c) 70%
d) 60%
e) 50%

46. (Ita 2016) Considere a expansão de um gás ideal inicialmente contido em um recipiente de
1L sob pressão de 10 atm. O processo de expansão pode ser realizado de duas maneiras
diferentes, ambas à temperatura constante:

I. Expansão em uma etapa, contra a pressão externa constante de 1 atm , levando o volume
final do recipiente a 10 L.
II. Expansão em duas etapas: na primeira, o gás expande contra a pressão externa constante
de 1 atm até atingir um volume de 2 L; na segunda etapa, o gás expande contra uma
pressão constante de 1 atm atingindo o volume final de 10 L.

Com base nestas informações, assinale a proposição CORRETA.


a) O trabalho realizado pelo gás é igual nos dois processos de expansão.
b) O trabalho realizado no primeiro processo é metade do trabalho realizado no segundo
processo.
c) A variação da energia interna do gás é igual em ambos os processos.
d) A variação da energia interna do gás no primeiro processo é metade da do segundo
processo.
e) O calor trocado pelo gás é igual em ambos os processos.

47. (Unicamp 2016) De modo simplificado, pode-se dizer que o parâmetro VO2 máximo
representa a capacidade orgânica máxima de um indivíduo absorver, transportar e utilizar o

Página 22 de 105
Interbits – SuperPro ® Web

oxigênio do ar atmosférico para a produção de energia via aeróbia. Esse parâmetro pode ser
informado para um indivíduo como um todo (mL O2 min) ou por massa corporal
(mL O2 kg min). O gráfico a seguir mostra valores médios de VO2 máximo para várias faixas
etárias, para homens (H) e mulheres (M), ativos (A) e sedentários (S). As faixas etárias são: 1
(15 a 24 anos), 2 (25 a 34 anos), 3 (35 a 44 anos), 4 (45 a 54 anos), 5 (55 a 64 anos) e 6 (65 a
74 anos).

a) Na maioria das competições esportivas, homens e mulheres são separados por se


considerar que eles não competiriam em igualdade. No entanto, de acordo com as
informações fornecidas, existiria alguma condição em que homens e mulheres teriam
mesma capacidade orgânica máxima de absorver, transportar e utilizar o oxigênio do ar
atmosférico, por massa corporal, para a produção da energia via aeróbia? Justifique.
b) Considere uma mulher ativa, que pesa 58 kg e que se encontra na faixa etária 4. De acordo
com a figura, se essa mulher se exercitar em seu VO2 máximo, ao final de uma hora
quantos gramas de gás oxigênio ela terá utilizado? Considere o volume molar do oxigênio
igual a 25 L mol−1.

48. (Ime 2016) Considere um dispositivo constituído por dois balões de vidro, " A" e "B ", cada
um com capacidade de 894 mL, interligados por um tubo de volume interno desprezível,
munido de uma torneira. Dois ensaios independentes foram realizados a 298 K. No primeiro
ensaio, os balões foram inicialmente evacuados e, logo a seguir, com a torneira fechada, foram
introduzidos 0,30 g de benzeno e 20,0 g de tolueno em " A" e "B ", respectivamente, de
modo que não houvesse contato entre as duas substâncias. No segundo ensaio, os balões
foram novamente evacuados e, na sequência, uma quantidade de benzeno foi introduzida em
" A" e outra quantidade de tolueno foi introduzida em "B". Considerando o comportamento
ideal para os gases e para as misturas, atenda aos seguintes pedidos:

a) determine a pressão em cada balão, no primeiro ensaio, após o sistema ter atingido o
equilíbrio;
b) uma vez aberta a torneira no segundo ensaio, calcule as frações molares de benzeno e
tolueno na fase gasosa no interior dos balões no momento em que o equilíbrio líquido-vapor
é atingido. Um manômetro acoplado ao dispositivo indica, nesse momento, uma pressão
interna de 76,2 mmHg.

49. (Ita 2016) Considerando um gás monoatômico ideal, assinale a opção que contém o
gráfico que melhor representa como a energia cinética média (Ec ) das partículas que compões

Página 23 de 105
Interbits – SuperPro ® Web

este gás varia em função da temperatura absoluta (T) deste gás.

a)

b)

c)

d)

e)

50. (Ita 2016) Dadas as informações:

I. O poder calorífico de um combustível representa a quantidade de calor gerada na combustão


por unidade de massa.
II. O poder calorífico do H2 (g) é aproximadamente 3 vezes o da gasolina.
III. O calor latente de ebulição do H2 ( ) é desprezível frente ao poder calorífico do H2 (g).
IV. A massa específica do H2 ( ) é de 0,071 g  cm−3 e a da gasolina é de 0,740 g  cm−3 .

Com base nestas informações, determine o valor numérico:

Página 24 de 105
Interbits – SuperPro ® Web

a) da massa de 45 L de gasolina.
b) do volume de H2 ( ) que, ao sofrer combustão, fornece a mesma quantidade de calor
liberada na combustão de 45 L de gasolina.
c) do volume que o H2 ocuparia se estivesse na forma de gás, a pressão de 1 bar e a 25 C.

51. (Unicamp 2016) A 2,5-dimetoxi-4-bromoanfetamina, DOB, é um potente alucinógeno


comercializado dentro de cápsulas, em doses de 1,5 mg. Essa quantidade é tão pequena que a
droga é conhecida como “cápsula do vento” ou “cápsula da morte”. A literatura não traz
informações sobre valores de dose letal, mas a ingestão de duas cápsulas da droga tem
grandes chances de levar o usuário a uma overdose.

a) Se o volume interno da cápsula em que se comercializa a droga é de 1,0 cm3 , quanto vale a
relação mDOB mar no interior da cápsula? Considere desprezível o volume ocupado pelo
DOB sólido, considere a pressão interna de 100.000 Pa e a temperatura de 25 C.
b) Imagine que um indivíduo ingere uma cápsula contendo 1,5 mg de DOB, ao mesmo tempo
em que outro indivíduo ingere um comprimido contendo 10 mg de ecstasy. Baseando-se
apenas no fato de que a meia-vida do DOB no organismo é de 12 horas e a do ecstasy é de
1,5 horas (uma hora e meia), qual dos dois indivíduos teria maior massa do princípio
ativo da droga após 12 horas? Na figura apresentada no espaço abaixo, construa as
curvas de decaimento das duas drogas no organismo para justificar sua resposta.

Dados: mDOB (massa de DOB); mar (massa de ar no interior da cápsula); massa molar do ar
= 29 g mol−1, R = 8,3 Pa m3 K −1 mol−1, T K = 273 + t C.

52. (Mackenzie 2016) Uma mistura gasosa ideal não reagente, formada por 10 g de gás
hidrogênio, 10 g de gás hélio e 70 g de gás nitrogênio encontra-se acondicionada em um
balão de volume igual a 5 L, sob temperatura de 27  C. A respeito dessa mistura gasosa, é
correto afirmar que

Dados:
- massas molares (g mol−1 ) H = 1, He = 4 e N = 14
- constante universal dos gases ideais (R) = 0,082 atm L mol−1 K −1
a) há, na mistura, 10 mol de gás hidrogênio, 2,5 mol de gás hélio e 5 mol de gás nitrogênio.
b) o gás nitrogênio exerce a maior pressão parcial dentre os gases existentes na mistura.
c) a pressão total exercida pela mistura gasosa é de 20 atm.
d) a fração em mols do gás hélio é de 25%.
e) o volume parcial do gás hidrogênio é de 2 L.

Página 25 de 105
Interbits – SuperPro ® Web

53. (Ita 2016) Uma amostra de 4,4 g de um gás ocupa um volume de 3,1L a 10 C e
566 mmHg. Assinale a alternativa que apresenta a razão entre as massas específicas deste
gás e a do hidrogênio gasoso nas mesmas condições de pressão e temperatura.
a) 2,2
b) 4,4
c) 10
d) 22
e) 44

54. (Ita 2015) Contribuíram de forma direta para o desenvolvimento do conceito de pressão
atmosférica
a) Friedrich August Kekulé e John Dalton.
b) Michael Faraday e Fritz Haber.
c) Galileu Galilei e Evangelista Torricelli.
d) Jöhns Jacob Berzelius e Eduard Büchner.
e) Robert Bunsen e Henry Louis Le Chatelier.

55. (Unicamp 2015) Notícia 1- Vazamento de gás oxigênio nas dependências do Hospital e
Maternidade São Mateus, Cuiabá, em 03/12/13. Uma empresária que atua no setor de venda
de oxigênio disse ao Gazeta Digital que o gás não faz mal para a saúde. “Pelo contrário, faz é
bem, pois é ar puro...”.
Adaptado de http://www.gazetadigital.com.br/conteudo/show/secao/9/materia/405285.
Acessado em 10/09/2014.

Notícia 2- Vazamento de oxigênio durante um abastecimento ao pronto-socorro da Freguesia


do Ó, zona norte de São Paulo, em 25/08/14. Segundo testemunhas, o gás que vazou do
caminhão formou uma névoa rente ao chão. O primeiro carro que pegou fogo estava ligado. Ao
ver o incêndio, os motoristas de outros carros foram retirar os veículos...
Adaptado de http://noticias.r7.com/sao-paulo/cerca-de-40-pacientes-sao-transferidos-apos-
incendio-em-hospital-da-zona-norte-26082014. Acessado em 10/09/2014.

Ficha de informações de segurança de uma empresa que comercializa esse produto.

a) Levando em conta as informações fornecidas na questão, você concorda ou discorda da


declaração da empresária na notícia 1? Justifique sua resposta.

b) Após o vazamento descrito na notícia 2, motoristas tentaram retirar os carros parados, mas
não tiveram êxito na sua tentativa. Qual deve ter sido a estratégia utilizada para que eles não
tenham tido êxito? Justifique, do ponto de vista químico, a razão pela qual não deveriam ter
utilizado essa estratégia.

56. (Upe 2014) A formulação de um determinado produto comercial contém, em massa, 58%
de solvente e 40% de uma mistura gasosa formada por CH3(CH2)2CH3, (CH3)2CHCH3 e
CH3CH2CH3, numa proporção de 65%, 15% e 20%, respectivamente.
Qual alternativa apresenta o produto que atende à descrição acima?
a) Desodorante aerossol
b) Extintor de incêndio
c) Gás de cozinha
d) Gás natural veicular – GNV
e) Gás refrigerante de geladeira

Página 26 de 105
Interbits – SuperPro ® Web

57. (Ita 2014) Considere um mol de um gás que se comporta idealmente, contido em um
cilindro indeformável provido de pistão de massa desprezível, que se move sem atrito. Com
relação a este sistema, são feitas as seguintes afirmações:

I. Se o gás for resfriado contra pressão externa constante, o sistema contrai-se.


II. Se pressão for exercida sobre o pistão, a velocidade média das moléculas do gás aumenta.
III. Se o sistema for aquecido a volume constante, a velocidade média das moléculas aumenta,
independentemente da natureza do gás.
IV. A velocidade média das moléculas será maior se o gás for o xenônio e menor se for o
argônio.

Das afirmações acima, está(ão) ERRADA(S) apenas


a) I e II.
b) I, III e IV.
c) II e III.
d) II e IV.
e) IV.

58. (Ime 2013) Um tambor selado contém ar seco e uma quantidade muito pequena de
acetona líquida em equilíbrio dinâmico com a fase vapor. A pressão parcial da acetona é de
180,0 mmHg e a pressão total no tambor é de 760,0 mmHg.
Em uma queda durante seu transporte, o tambor foi danificado e seu volume interno diminuiu
para 80% do volume inicial, sem que tenha havido vazamento. Considerando-se que a
temperatura tenha se mantido estável a 20°C, conclui-se que a pressão total após a queda é
de:
a) 950,0 mmHg
b) 1175,0 mmHg
c) 760,0 mmHg
d) 832,0 mmHg
e) 905,0 mmHg

59. (Ime 2013) Um gás possui uma taxa de efusão que corresponde a 25,0% da taxa do gás
hidrogênio. Uma massa mx desse gás, que ocupa um volume de 1,00 L a 1,00 atm e a 39,5°C,
é a mesma de sulfanilamida, um soluto não volátil, dissolvida em 100 g de acetona. Se a
pressão de vapor da acetona pura a 39,5°C é 400 mmHg, calcule:
a) a massa mx; e
b) a pressão de vapor da solução de sulfanilamida (C6H8O2N2S) em acetona à mesma
temperatura.

Dado: R = 0,082 atm.L.mol –1.K –1

60. (Ime 2012) Um volume V1 de oxigênio e um volume V2 de ácido sulfídrico, ambos nas
mesmas condições de temperatura e pressão, são misturados. Promovendo-se a reação
completa, verifica-se que os produtos da reação, quando colocados nas condições iniciais de
pressão e temperatura, ocupam um volume de 10 L.
Considere que a água formada encontra-se no estado líquido e que as solubilidades dos gases
em água são desprezíveis. Sabendo-se que havia oxigênio em excesso na reação e que
V1 + V2 = 24 L, verifica-se que o valor de V2 é:
a) 14,7 L
b) 9,3 L
c) 12,0 L
d) 5,7 L
e) 15,7 L

61. (Fuvest 2012)

Página 27 de 105
Interbits – SuperPro ® Web

A um recipiente, contendo solução aquosa de ácido sulfúrico, foi adicionada uma massa m de
carbonato de sódio. Imediatamente após a adição desse sal, foi adaptado, à boca do
recipiente, um cilindro de raio r, no interior do qual um êmbolo, de massa desprezível, pode se
deslocar sem atrito. Após algum tempo, o carbonato de sódio foi totalmente consumido, e o gás
liberado moveu o êmbolo para cima. Nessa transformação, o ácido sulfúrico era o reagente em
excesso.

a) Escreva a equação química balanceada que representa a transformação que ocorreu dentro
do recipiente.
b) O experimento descrito foi repetido utilizando-se carbonato de potássio em lugar de
carbonato de sódio. A massa de carbonato de potássio utilizada nesse segundo experimento
também foi m. A altura atingida pelo êmbolo foi a mesma nos dois experimentos? Explique.
(Considere desprezível a variação de temperatura no sistema).
c) Escreva a expressão matemática que relaciona a altura x, atingida pelo êmbolo, com a
massa m de carbonato de sódio. Para isso, considere que
- a solubilidade do gás, na solução, é desprezível, e não há perda de gás para a atmosfera;
- nas condições do experimento, o gás formado se comporta como um gás ideal, cujo
volume é dado por V = nRT/P, em que:
P = pressão do gás
n = quantidade de matéria do gás (em mol)
R = constante universal dos gases
T = temperatura do gás (em K)

Observação: Use a abreviatura MM para representar a massa molar do carbonato de sódio.

62. (Ime 2012) A determinada profundidade, o organismo de um mergulhador absorve N 2 a


uma pressão parcial de 5,0 atm. Considere que a solubilidade do N 2 no sangue, a uma pressão
parcial de 0,78 atm, seja 5,85  10 −4 mol/L. Admita, ainda, que o volume total de sangue no
corpo do mergulhador possa ser estimado em 6,0 L. Nessas condições, estima-se que a
quantidade de N2, em mol, que o mergulhador elimina em seu retorno à superfície, onde a
pressão parcial desse gás é 0,78 atm, seja:
a) 3,50  10−3
b) 7,30  10−3
c) 1,90  10 −2
d) 1,21 10 −2
e) 2,25  10−2

Página 28 de 105
Interbits – SuperPro ® Web

63. (Unicamp 2012) Recentemente a Prefeitura de São Paulo ameaçava fechar as portas de
um centro comercial por causa do excesso de gás metano em seu subsolo. O empreendimento
foi construído nos anos 1980 sobre um lixão e, segundo a CETESB, o gás metano poderia
subir à superfície e, eventualmente, causar explosões.

a) Uma propriedade que garante a ascensão do metano na atmosfera é a sua densidade.


Considerando que os gases se comportam como ideais, e que a massa molar média do ar
atmosférico é de 28,8 g mol-1, justifique esse comportamento do metano em relação ao ar
atmosférico.
b) Na época do acontecimento, veiculou-se na imprensa que, “numa mistura com o ar, se o
metano se encontra dentro de um determinado percentual (5% a 15% em volume quando
em ar ambiente com 21% de oxigênio) e existe uma faísca ou iniciador, a explosão irá
ocorrer”. Partindo-se do ar atmosférico e de metano gasoso, seria possível obter a mistura
com a composição acima mencionada, pela simples mistura desses gases? Justifique.

64. (Ime 2012) Considere a reação

AB2 ( g) + A ( s ) 2 AB ( g )

Atingido o equilíbrio nas CNTP, a fase gasosa apresenta fração molar de AB2 igual a 0,1. Em
que pressão, à mesma temperatura, a fração molar de AB na fase gasosa, no equilíbrio, seria
igual a 0,8?

65. (Upe 2012) Dois chumaços de algodão, I e II, embebidos com soluções de ácido clorídrico,
HC , e amônia, NH3, respectivamente, são colocados nas extremidades de um tubo de vidro
mantido fixo na horizontal por um suporte, conforme representação abaixo. Após um certo
tempo, um anel branco, III, forma-se próximo ao chumaço de algodão I.

Baseando-se nessas informações e no esquema experimental, analise as seguintes


afirmações:

I. O anel branco forma-se mais próximo do HC , porque este é um ácido forte, e NH3 é uma
base fraca.
II. O anel branco formado é o NH4 C sólido, resultado da reação química entre HC e NH3
gasosos.
III. O HC é um gás mais leve que NH3, logo se movimenta mais lentamente, por isso o anel
branco está mais próximo do ácido clorídrico.

Está correto o que se afirma em


Dados: massas molares, H = 1g/mol; C = 35,5 g/mol; N = 14 g/mol.
a) II.
b) III.
c) I e II.
d) I e III.
e) II e III.

66. (Fuvest 2012)

Página 29 de 105
Interbits – SuperPro ® Web

Uma estudante de Química realizou um experimento para investigar as velocidades de difusão


dos gases HC e NH3. Para tanto, colocou, simultaneamente, dois chumaços de algodão nas
extremidades de um tubo de vidro, como mostrado na figura acima. Um dos chumaços estava
embebido de solução aquosa de HC (g), e o outro, de solução aquosa de NH3(g). Cada um
desses chumaços liberou o respectivo gás. No ponto de encontro dos gases, dentro do tubo,
formou-se, após 10 s, um anel de sólido branco ( NH4 C ), distante 6,0 cm do chumaço que
liberava HC (g).

a) Qual dos dois gases, desse experimento, tem maior velocidade de difusão? Explique.
b) Quando o experimento foi repetido a uma temperatura mais alta, o anel de NH4 C (s) se
formou na mesma posição. O tempo necessário para a formação do anel, a essa nova
temperatura, foi igual a, maior ou menor do que 10 s? Justifique.
c) Com os dados do experimento descrito, e sabendo-se a massa molar de um dos dois gases,
pode-se determinar a massa molar do outro. Para isso, utiliza-se a expressão
velocidade de difusão do NH3 (g) massa molar do HC
=
velocidade de difusão do HC (g) massa molar do NH3
Considere que se queira determinar a massa molar do HC . Caso o algodão embebido de
solução aquosa de NH3 (g) seja colocado no tubo um pouco antes do algodão que libera
HC (g) (e não simultaneamente), como isso afetará o valor obtido para a massa molar do
HC ? Explique.

67. (Ime 2012) Na figura, uma solução concentrada de HC , contida em A, é gotejada sobre
zinco sólido em B. Um dos produtos dessa reação escoa para C, onde é completamente
consumido na reação com o vapor de uma substância simples, cujo elemento pertence à
família 17. O produto da reação ocorrida em C é um gás incolor. A válvula V permite somente o
escoamento no sentido de B para C. O recipiente C possui volume de 1,0 L, é mantido a 100°C
durante todo o processo e contém inicialmente 0,05 mol da substância simples supracitada.

Observações:
— os volumes das conexões e tubulações devem ser desconsiderados;
— a substância presente inicialmente em C é um líquido marrom-avermelhado à temperatura
ambiente.

Página 30 de 105
Interbits – SuperPro ® Web

Determine:
a) as reações que ocorrem em B e C, identificando o estado físico de cada uma das
substâncias envolvidas.
b) o número máximo de mols do produto da reação em B que pode escoar para C, sem que a
pressão neste exceda 2,0 atm, se a extremidade D for fechada.

68. (Fuvest 2011) Maçaricos são queimadores de gás utilizados para produzir chamas de
elevadas temperaturas, como as requeridas para soldar metais. Um gás combustível, muito
utilizado em maçaricos, é o acetileno, C2H2, sendo que a sua combustão pode ser promovida
com ar atmosférico ou com oxigênio puro.

a) Escreva a equação química balanceada da combustão completa do acetileno com oxigênio


puro.
b) Em uma oficina de solda, existem dois cilindros idênticos, um deles contendo oxigênio puro
(cilindro A) e o outro, ar atmosférico (cilindro B). Sabendo que, no interior dos dois cilindros,
as condições de pressão e temperatura são as mesmas, qual dos dois cilindros contém a
maior massa gasosa? Explique.
c) A temperatura da chama do maçarico é maior quando se utiliza a mistura de oxigênio e
acetileno do que quando se usa a mistura de ar atmosférico e acetileno, mesmo estando os
reagentes em proporção estequiométrica nos dois casos. Considerando as substâncias
gasosas que recebem o calor liberado na combustão, em cada caso, explique essa diferença
de temperatura.

massa molar
g mol-1
O2 32
N2 28

69. (Ime 2011) Um gás ideal sofre uma mudança de estado ilustrada pelos gráficos I e II
abaixo.

Página 31 de 105
Interbits – SuperPro ® Web

Dentre as alternativas abaixo, assinale aquela que se ajusta aos gráficos acima.
a) α é o volume, β é a temperatura, δ é a pressão e o processo é uma expansão a
temperatura constante.
b) δ é a temperatura, β é a pressão, α é o volume e o processo é uma compressão.
c) α é o volume, β é a pressão, δ é a temperatura e o processo é um resfriamento isobárico.
d) α é o volume, β é a temperatura, δ é a pressão e o processo é uma compressão
isotérmica.
e) α é a pressão, β é o volume, δ é a temperatura e o processo é um aquecimento isobárico.

70. (Ita 2011) Um sistema em equilíbrio e composto por n0 mol de um gás ideal a pressão P0,
volume V0, temperatura T0 e energia interna U0. Partindo sempre deste sistema em equilíbrio,
são realizados isoladamente os seguintes processos:

I. Processo isobárico de T0 até T0/2.


II. Processo isobárico de V0 até 2V0.
III. Processo isocórico de P0 até P0/2.
IV. Processo isocórico de T0 até 2T0.
V. Processo isotérmico de P0 até P0/2.
VI. Processo isotérmico de V0 até V0/2.

Admitindo que uma nova condição de equilíbrio para esse sistema seja atingida em cada
processo x (x = I, II, III, IV, V e VI), assinale a opção que contem a informação errada.
a) UV = UVI/2
b) UVI = U0
c) PIV = PVI
d) TII = 4TIII
e) VI = VV/4

71. (Ime 2011) Um recipiente de paredes rígidas, contendo apenas ar, aberto para a
atmosfera, é aquecido de 27°C a 127°C. Calcule a percentagem mássica de ar que saiu do
recipiente, quando atingido o equilíbrio final.
a) 79%
b) 75%
c) 30%
d) 25%
e) 21%

72. (Fuvest 2011) Um laboratório químico descartou um frasco de éter, sem perceber que, em
seu interior, havia ainda um resíduo de 7,4 g de éter, parte no estado líquido, parte no estado
gasoso. Esse frasco, de 0,8 L de volume, fechado hermeticamente, foi deixado sob o sol e,
após um certo tempo, atingiu a temperatura de equilíbrio T = 37 ºC, valor acima da temperatura
de ebulição do éter. Se todo o éter no estado líquido tivesse evaporado, a pressão dentro do
frasco seria

Página 32 de 105
Interbits – SuperPro ® Web

NOTE E ADOTE
No interior do frasco descartado havia apenas éter.
Massa molar do éter = 74 g
K = ºC + 273
R (constante universal dos gases) = 0,08 atm.L / (mol.K)
a) 0,37 atm.
b) 1,0 atm.
c) 2,5 atm.
d) 3,1 atm.
e) 5,9 atm.

73. (Ita 2011) Considere dois cilindros idênticos (C1 e C2), de paredes rígidas e indeformáveis,
inicialmente evacuados. Os cilindros C1 e C2 são preenchidos, respectivamente, com O 2(g) e
Ne(g) até atingirem a pressão de 0,5 atm e temperatura de 50°C. Supondo comportamento
ideal dos gases, são feitas as seguintes afirmações:
I. O cilindro C1 contém maior quantidade de matéria que o cilindro C2.
II. A velocidade média das moléculas no cilindro C1 e maior que no cilindro C2 .
III. A densidade do gás no cilindro C1 é maior que a densidade do gás no cilindro C2.
IV. A distribuição de velocidades das moléculas contidas no cilindro C1 e maior que a das
contidas no cilindro C2.

Assinale a opção que apresenta a(s) afirmação(ões) correta(s).


a) Apenas I e III.
b) Apenas I e IV.
c) Apenas II.
d) Apenas II e IV.
e) Apenas III.

74. (Ita 2011) O dióxido de carbono representa, em média, 0,037% da composição volumétrica
do ar seco atmosférico, nas condições ambientes. Esse gás, dissolvido em água, sofre um
processo de hidratação para formar um ácido diprótico, que se ioniza parcialmente no líquido.
Admitindo-se que água pura seja exposta a CO2 (g) atmosférico, nas condições ambientes, e
sabendo que o equilíbrio entre as fases gasosa e líquida desse gás é descrito pela lei de
Henry, calcule:

a) a solubilidade do CO2 (aq), expressa em mg L–1, nas condições especificadas acima,


sabendo que a constante da lei de Henry para CO2 gasoso dissolvido em água a 25 °C é 3,4
x 10–2 mol L–1 atm–1.
b) a concentração molar do ânion bicarbonato, expressa em mol L–1, sabendo que a constante
de dissociação ácida para o primeiro equilíbrio de ionização do ácido diprótico a 25 °C é 4,4
x 10–7.

75. (Ime 2010) Em um recipiente fechado queima-se propano com 80% da quantidade
estequiométrica de ar. Admitindo que não haja hidrocarbonetos após a combustão, que todos
os produtos da reação estejam na fase gasosa e que a composição volumétrica do ar seja de
uma parte de O2 para quatro partes de N2 , calcule a porcentagem molar de CO2 no recipiente
após a combustão (considere comportamento ideal para os gases).
a) 4,35 %
b) 4,76 %
c) 5,26 %
d) 8,70 %
e) 14,28 %

76. (Ime 2010) Um sistema fechado e sem fronteiras móveis contém uma determinada massa
gasosa inerte. Sabe-se que, após aquecimento, o sistema registra um aumento de 5% na
pressão e de 15 ºC na temperatura (considere que o gás se comporta idealmente). A respeito
do valor da temperatura inicial, pode-se dizer que:
a) é igual ou inferior a 30 ºC.
b) é superior a 30 ºC e inferior a 300 ºC.
c) é igual ou superior a 300 ºC.

Página 33 de 105
Interbits – SuperPro ® Web

d) somente pode ser calculado conhecendo-se o volume e a massa de gás.


e) somente pode ser calculado conhecendo-se o volume, a massa e a pressão inicial do gás.

77. (Ime 2010) As alternativas abaixo representam processos hipotéticos envolvendo 2 mols
de um gás ideal, contidos em um conjunto cilindro-pistão. Assinale a alternativa que apresenta
mais de três estados (V, T) nos quais a pressão é máxima:

a)

b)

c)

d)

e)

TEXTO PARA AS PRÓXIMAS 3 QUESTÕES:


Massas atômicas (u.m.a)

O C H N Zn
16 12 1 14 65,4

In 2 = 0,69.
Lei de decaimento radioativo: N = N0 e−kt
Constante criométrica da água = 2 K  kg  mol−1
Massa específica da água = 1,0 g/mL

Página 34 de 105
Interbits – SuperPro ® Web

R = 0,082 atm  L  mol−1  K −1 = 8,314 J  mol−1K −1

78. (Ime 2010) O brometo de alquila X, opticamente ativo, é tratado com brometo de etil-
magnésio, gerando-se o composto Y. A 100°C, 8,4 g de Y no estado gasoso são misturados
com 6,4 g de N2 em um recipiente com volume de 2,0 litros. A pressão medida no interior do
recipiente é de 5,0 atm.
Considerando que os gases se comportam idealmente, determine as fórmulas estruturais
planas e a nomenclatura IUPAC dos compostos X e Y. Justifique a sua solução.

79. (Ime 2010) Uma massa de óxido ferroso é aquecida a 1273 K e, em seguida, exposta a
uma mistura gasosa de monóxido de carbono e hidrogênio. Desta forma, o óxido é reduzido a
metal sem qualquer fornecimento adicional de energia. Admita que ocorra uma perda de calor
para as circunvizinhanças de 4,2 kJ/mol de óxido reduzido. Calcule a razão mínima entre as
(
pressões parciais de monóxido de carbono e de hidrogênio PCO PH2 ) na mistura gasosa
inicial, de modo que o processo seja autossustentável. Despreze a decomposição da água.

Calores de reação a 1273 K (kJ/mol):


Redução do óxido ferroso 265
Oxidação do hidrogênio - 250
Oxidação do monóxido de carbono - 282

80. (Ime 2010) A transformação isovolumétrica de um gás triatômico hipotético A 3 em outro


diatômico A 2 envolve a liberação de 54 kJ/mol de A 3 . A capacidade calorífica molar, a
volume constante do gás A 2 , é de 30 J/mol.K. Após a transformação isocórica de todo A 3 em
A 2 , determine o aumento percentual de pressão em um recipiente isolado contendo o gás A 3
a 27°C.
Considere que a capacidade calorífica molar, a volume constante do gás A 2 , não varia com a
temperatura e que os gases se comportam idealmente.

81. (Fuvest 1999) Certo refrigerante é engarrafado, saturado com dióxido de carbono (CO2 ) a
5 C e 1 atm de CO2 e então fechado. Um litro desse refrigerante foi mantido algum tempo
em ambiente à temperatura de 30 C. Em seguida, a garrafa foi aberta ao ar (pressão
atmosférica = 1 atm) e agitada até praticamente todo CO2 sair. Nessas condições (30 C e
1 atm), qual o volume aproximado de CO2 liberado?

Dados:
massa molar de CO2 = 44 g mol
volume molar dos gases a 1 atm e 30C = 25 L mol
solubilidade do CO2 no refrigerante a 5 C e sob 1 atm de CO2 = 3,0 g L
a) 0,40 L
b) 0,85 L
c) 1,7 L
d) 3,0 L
e) 4,0 L

82. (Ita 1995) Em três frascos rotulados A, B e C e contendo 100 ml de água cada um, são
colocados 0,1 mol, respectivamente, de hidróxido de potássio, hidróxido de cobre (II) e
hidróxido de níquel (II). Após agitar o suficiente para garantir que todo soluto possível de se
dissolver já esteja dissolvido, mede-se as condutividades elétricas das misturas. Obtém-se que

Página 35 de 105
Interbits – SuperPro ® Web

as condutividades das misturas dos frascos B e C são semelhantes e muito menores do que a
do frasco A.

Assinale a opção que contém a afirmação FALSA.


a) Nos frascos B e C, a parte do hidróxido que está dissolvida encontra-se dissociada
ionicamente.
b) Os hidróxidos dos copos B e C são bases fracas, porque nem toda quantidade dissolvida
está dissociada ionicamente.
c) A condutividade elétrica da mistura do frasco A é a maior porque se trata de uma solução 1
molar de eletrólito forte.
d) Os três solutos são bases fortes, porém os hidróxidos de cobre (II) e de níquel (II) são pouco
solúveis.
e) Soluções muito diluídas com igual concentração normal destes 3 hidróxidos deveriam
apresentar condutividades elétricas semelhantes.

83. (Ime 2015) Tomou-se uma amostra de 130 g de zinco metálico para reagir com uma
solução aquosa diluída de ácido clorídrico em quantidade estequiométrica. Dessa reação,
observou-se a formação de gás, que foi aquecido a 227C e transportado para um balão
fechado de 50 L. Esse balão continha, inicialmente, iodo em fase gasosa a 227C e 3,28 atm.
Após o equilíbrio, verificou-se que a constante de equilíbrio K C a 227C é igual a 160.
Considerando que a temperatura permaneceu constante durante o processo, determine a
pressão final total no balão.

Página 36 de 105
Interbits – SuperPro ® Web

Gabarito:

Resposta da questão 1:
[E]

De acordo com o texto do enunciado da questão, a primeira lei da termodinâmica, associa a


variação de energia interna à diferença entre o calor trocado com o meio exterior e o trabalho
realizado no processo. Então:
ΔU : var iação de energia int erna
Q : calor
Q = 0 (expansão adiabática; sem troca de calor)
W : trabalho
W  0 (expansão)
ΔU = Q − W
ΔU = 0 − ( + W )
ΔU = − W

Conclusão: o trabalho é associado diretamente à variação de energia interna e não há troca de


calor entre o gás e o ambiente.

Resposta da questão 2:
[D]

T = 7 C + 273 = 280K
V = 24 mL = 24  10 −6 m3
1Pa = 1N  m−2
P = 2,8  105 Pa = 2,8  105 N  m −2
1J = Nm
R = 8 J  K −1  mol−1 = 8 N  m  K −1  mol−1

Aplicando a equação de estado de gases ideais, vem:


P V = nR T
2,8  105 N  m−2  24  10−6 m3 = n  8 N  m  K −1  mol−1  280 K
2,8  105 N  m−2  24  10 −6 m3
n= = 0,03  105  10−6 mol
−1 −1
8 Nm K  mol  280 K
−3
n = 3,0  10 mol

Resposta da questão 3:
a) Com base no esquema, o N2( ) foi utilizado para causar a sublimação do gás carbônico
sublimação
(CO2(g) ⎯⎯⎯⎯⎯⎯
→ CO2(s) ), pois sua temperatura de fusão (mudança do estado de
agregação líquido para sólido e vice-versa) é menor do que a do CO2. Assim, pode ocorrer a
transferência de calor necessária ao processo.

b) Grafite (menos compacto):

Página 37 de 105
Interbits – SuperPro ® Web

Diamante (mais compacto):

Para um valor de massa constante (padrão), o volume ocupado pelo grafite é maior, logo
sua densidade é menor do que a do diamante.
( ) (
dGrafite 2,0 g cm3  dDiamante 3,5 g cm3 )
Utilizando-se o mesmo volume e substituindo-se diamante por grafite, a quantidade de CO2
formada ao final do processo será menor, pois a massa de carbono por cm3 de material no
grafite é menor do que no diamante.

c) Cálculo da massa de gelo CO2(s) ( ) seco obtida:


C = 12; CO2 = 1 12 + 2  16 = 44
1Cd + 1O2(g) ⎯⎯→ 1CO2(s)
12 g 44 g
3g mCO2(s)
3 g  44 g
mCO2(s) = = 11 g
12 g

Cálculo da pressão gerada pelo CO2(g) dentro do cilindro de gás a 27 °C:


mCO2 = 11 g; MCO2 = 44 g  mol−1
T = 27 + 273 = 300 K
R = 0,082 atm  L mol−1  K −1
Vint erno = 1 L
mCO2
P  Vint erno = nCO2  R  T  P  Vint erno = R T
MCO2
11 g
P 1 L =  0,082 atm  L mol−1  K −1 300 K
44 g  mol−1
P = 6,15 atm

Resposta da questão 4:
[B]

Combustíveis de interesse: H2, CH4 e C2H4 .

Página 38 de 105
Interbits – SuperPro ® Web

Hidrocarbonetos de interesse:
CH4 = 1 12 + 4  1 = 16; MCH4 = 16 g  mol−1

C2H4 = 2  12 + 4  1 = 28; MC2H4 = 28 g  mol−1

CH4 : hidrocarboneto de baixa massa (leve); MCH4 = 16 g  mol−1

C2H4 : hidrocarboneto de baixa massa (leve); MCH4 = 28 g  mol−1

Compostos que não geraram interesse para serem utilizados como combustível:
CO : móxido de carbono (óxido)
CH3 CHO : aldeído (E tanal)
CO2 : dióxido de carbono (óxido)

Conclusão: o objetivo foi alcançado para hidrocarbonetos leves.

Como, nas mesmas condições de pressão e temperatura, o volume é diretamente proporcional


ao número de mols (hipótese de Avogadro), constata-se pela análise da figura que a maior
quantidade em mol de gás obtida foi a de monóxido de carbono (CO), um composto que não
gerou interesse para ser utilizado como combustível.

Resposta da questão 5:

Página 39 de 105
Interbits – SuperPro ® Web

nN2 = 0,79  nar (I)  79


 (I)  (II)  nN2 =  nO2
nO2 = 0,21 nar (II) 21
C3H8 = 3  12 + 8  1 = 44; C4H10 = 4  12 + 10  1 = 58; O2 = 2  16 = 32; N2 = 2  14 = 28
Combustão
do propano
C3H8 + 5O2 ⎯⎯⎯⎯⎯⎯
→ 3CO2 + 4H2O
Combustão
do bu tano
C4H10 + 6,5O2 ⎯⎯⎯⎯⎯⎯ → 4CO2 + 5H2O
C3H8 + C4H10 + 11,5 O2 ⎯⎯→ 7CO2 + 9H2O
Mistura equimolar : 1000 mol (500 mol de C3H8 + 500 mol de C4H10 )
500C3H8 + 500C4H10 + 11,5  500 O2 ⎯⎯→ 7  500CO2 + 9  500H2O

( )
500C3H8 + 500C4H10 + 5750 + nO2 (excesso) O2 + nN2 ⎯⎯→ ...
nO
2

... ⎯⎯→ 3500CO2 + 4500H2O + nO2 (excesso) + nN2


Gases de exaustão

Página 40 de 105
Interbits – SuperPro ® Web

ngases (exaustão) = 3500 + 4500 + nO2 (excesso) + nN2


79
ngases (exaustão) = 8000 + nO2 (excesso) +  nO2
21
ngases (exaustão) = 8000 + nO2 (excesso) +
79
21
(
 5750 + nO2 (excesso) )
3
nO2 (excesso) =  ngases (exaustão)  nO2 (excesso) = 0,03  ngases (exaustão)
100

ngases (exaustão) = 0,03  8000 + nO2 (excesso) +

79
21
(
 5750 + nO2 (excesso) 


)
79 79
nO2 (excesso) = 0,03  8000 + 0,03nO2 (excesso) + 0,03   5750 + 0,03   nO2 (excesso)
21 21
240
648,929 0,1129
nO2 (excesso) − 0,03  nO2 (excesso) − 0,1129  nO2 (excesso) = 240 + 648,929
0,8571 nO2 (excesso) = 888,929
888,929
nO2 (excesso) = = 1037,13 mol
0,8571
nO2 (total) = 5750 + nO2 (excesso) = 5750 + 1037,13  nO2 (total) = 6787,13 mol
m = nM
mO2 (total) = 6787,13 mol  32 g  mol−1 = 217188,16 g
79 79
nN2 =  nO2 (total)  nN2 =  6787,13 mol = 25532,536 mol
21 21
mN2 = 25532,536 mol  28 g  mol−1 = 714911 g
mar = mO2 (total) + mN2
mar = 217188,16 g + 714911 g = 932099,16 g
mcombustível = mC3H8 + mC4H10  mcombustível = nC3H8  MC3H8 + nC4H10  MC4H10

MC3H8 = 44 g  mol−1; MC4H10 = 58 g  mol−1

mcombustível = 500 mol  44 g  mol−1 + 500 mol  58 g  mol −1 = 51000 g


mar 932099,16 g
Razão mássica = =
mcombustível 51000 g
Razão mássica = 18,276

Resposta da questão 6:
ANULADA

Questão anulada no gabarito oficial.

Motivo da anulação da questão: a equação do equilíbrio da amônia (NH3(g) ) com o


nitrogênio (N2(g) ) e hidrogênio (H2(g) ) não foi fornecida no texto da questão, assim poderia ser
suposta a decomposição da amônia ao invés da formação da mesma, ou a alteração dos
valores dos coeficientes da reação de formação da amônia (por exemplo: 12 , 3 2 , 1), o que
forneceria outras possibilidades de respostas.

Observações:
⎯⎯
Decomposição da amônia: 2NH3(g) ⎯→ 1N2(g) + 3H2(g) .

Página 41 de 105
Interbits – SuperPro ® Web

⎯⎯
Formação da amônia: 1N2(g) + 3H2(g) ⎯→ 2NH3(g) .

Alteração dos valores dos coeficientes (possível exemplo, entre vários):
⎯⎯
1NH3(g) ⎯→ 1 N2(g) + 3 H2(g) ou 1 N2(g) + 3 H2(g) ⎯
⎯ ⎯⎯→ 1NH3(g) .

2 2 2 2

De acordo com o enunciado da questão, a válvula do reator A se abre automaticamente


quando são produzidas em seu interior pressões iguais ou superiores a 1,5 atm. Neste reator
foi armazenada hidrazina líquida (N2H4 ) , que se decompôs inteiramente em 0,163 mol de gás
hidrogênio e 0,082 mol de gás nitrogênio a 819 oC.
Então:
Decomposição
1N2H4( ) ⎯⎯⎯⎯⎯⎯⎯
→ 2H2(g) + 1N2(g)
nH2 = 0,163 mol 
 n = nH2 + nN2 = ( 0,163 + 0,082 ) mol = 0,245 mol
nN2 = 0,082 mol
Tinicial = 819 C 
 TC = Tinicial + ΔT = ( 819 + 10 ) C = 829 C
ΔT = 10 C 
T = 829 + 273 = 1102 K
V = 20 L (reator A)
R = 0,082 atm  L  mol−1  K −1
nR  T
P V = nR T  P =
V
0,245 mol  0, 082 atm  L  mol−1  K −1  1102 K
P=
20 L
P = 1,107 atm  1,5 atm  Válvula A não abre, permanece fechada.

De acordo com o enunciado da questão, a válvula do reator B se abre automaticamente


quando são produzidas em seu interior pressões iguais ou superiores a 3,5 atm. Neste reator
encontram-se em equilíbrio, amônia, 1,03  10−2 mol L de N2 e 1,62  10−2 mol L de H2 , a 819
°C (1102 K), com um valor de Kp (constante de equilíbrio em termos de pressão parcial) igual a
0,25.

Supondo a escolha da reação de formação de amônia (padrão) com coeficientes 1, 3, 2:

Página 42 de 105
Interbits – SuperPro ® Web

N2  = 1,03  10−2 mol  L−1


pN2 = N2   R  T = 1,03  10 −2 mol  L−1  0,082 atm  L  mol−1  K −1  1102 K  pN2 = 0,93 atm

H2  = 1,62  10−2 mol  L−1


pH2 = N2   R  T = 1,62  10 −2 mol  L−1  0,082 atm  L  mol−1  K −1  1102 K  pH2 = 1, 46 atm

⎯⎯
1N2(g) + 3 H2(g) ⎯→ 2 NH3(g) KP = 0,25

(pNH ) (pNH )
2 2

KP = 3
 0,25 = 3

(pN )  (pH ) ( 0,93 )1  (1,46 )3


1 3
2 2

(pNH ) = 0,72
2
3

pNH3 = 0,72 = 22  32  2  10 −2

pNH3 = 0,6  2 = 0,85 atm


Ptotal = pN2 + pH2 + pNH3 = ( 0,93 + 1,46 + 0,85 ) atm
Ptotal = 3,24 atm  3,5 atm  Válvula B não abre, permanece fechada.

Conclusão (supondo a escolha da reação de formação de amônia (padrão) com coeficientes 1,


3, 2): alternativa [D]; as válvulas de ambos os reatores permanecem fechadas.

Resposta da questão 7:
[D]

R = 0,0821 L  atm  K −1  mol−1


T = 25 + 273 = 298 K
P = 1 atm
NaN3 = 65 g  mol−1
2NaN3(s) ⎯⎯→ 2Na(s) + 3 N2 (g)
2  65 g 3 mol
100 g n N2
100 g  3 mol 30
n N2 = = mol
2  65 g 13
P  VN2 = n N2  R  T
30
1 atm  VN2 = mol  0,0821 L  atm  K −1  mol−1  298 K
13
VN2 = 56,459 L = 56,5 L

Resposta da questão 8:
a) Os gases reagem entre si formando óxido nítrico (NO) e água (H 2O) no estado gasoso.
Equação química balanceada:
5 3
1NH3(g) + O2(g) ⎯⎯→ 1NO(g) + H2O(g) ou
4 2
4NH3(g) + 5O2(g) ⎯⎯→ 4NO(g) + 6H2O(g)

b) Cálculo dos valores numéricos das frações molares de amônia e de oxigênio no início da
reação:

Página 43 de 105
Interbits – SuperPro ® Web

NH3 = 1 14 + 3  1 = 17; MNH3 = 17 g  mol−1

O2 = 2  16 = 32; MO2 = 32 g  mol−1


XNH3 + XO2 = 1  XNH3 = 1 − XO2

Maparente = 29 g  mol−1
Maparente = MNH3  XNH3 + MO2  XO2
29 = 17  (1 − XO2 ) + 32  XO2
29 = 17 − 17  XO2 + 32  XO2
15  XO2 = 29 − 17
3
XO2 = = 0, 20 = 20 %
15
3
XNH3 = 1 − XO2  XNH3 = 1 −
15
12
XNH = = 0,80 = 80%
3 15

c) Cálculo do valor numérico da porcentagem de reagente em excesso:


nNH3 = 0,20n; nO2 = 0,80n

1NH3(g) + 1,25 O2(g) ⎯⎯→ 1NO(g) + 1,5H2O(g)


1 mol 1,25 mol
0,20n mol 0,80n
Com
excesso

(1 0,80n )  (1,25  0,20n )


1NH3(g) + 1,25 O2(g) ⎯⎯→ 1NO(g) + 1,5H2O(g)
1 mol 1,25 mol
0,20n mol nO2
0,20n mol 1,25 mol
nO = = 0,25n
2 1 mol
nExcesso = 0,80n − 0,25n = 0,55n
0,80n 100%
0,55n p
0,55n  100%
p= = 68,75%
0,80n

d) Cálculo dos valores numéricos das quantidades (em mols) das espécies químicas no final da
reação.
5 3
1NH3(g) + O2(g) ⎯⎯→ 1NO(g) + H2O(g)
4 2
20 80 0 0 (início; mol)
5 3
−20 −  20 20 +  20 (durante; mol)
4 2
0 55 20 30 (final; mol)
nNH3 = 0 mol; nO2 = 55 mol ; nNO = 20 mol; nH2O = 30 mol

Página 44 de 105
Interbits – SuperPro ® Web

Resposta da questão 9:
a) Equação química balanceada dessa reação de combustão do óxido nitroso (N 2O) com
dissulfeto de carbono (CS2) produzindo enxofre sólido (S8):
8N2O + 4CS2 ⎯⎯→ 1S8 + 8N2 + 4CO2

b) Determinação do valor numérico do volume de gás gerado (em litros):


CS2 = 1 12 + 2  32 = 76
MCS2 = 76 g  mol−1

P = 105 Pa
T = 300 K
1 J = Pa  m3
R = 8,31 J  K −1  mol−1 = 8,31 Pa  m3  K −1  mol−1
P  Vmolar = 1 R  T
RT
Vmolar =
P
8,31 Pa  m3  K −1  mol−1  300 K
Vmolar =
105 Pa
Vmolar = 0,02493 m3 = 24,93 L
8N2O + 4 CS2 ⎯⎯→ 1S8 + 8N2 + 4 CO2
8 + 4 = 12 mol
4  76 g 12  24,93 L
304 g V
304 g  12  24,93 L
V=
4  76 g
V = 299,16 L

c) Cálculo do valor numérico da massa de enxofre sólido (em g) gerado considerando a mesma
quantidade de dissulfeto de carbono do item (b):
8N2O + 4 CS2 ⎯⎯⎯
→ 1S8 + 8N2 + 4 CO2
4  76 g 8  32 g
304 g mS
8
304 g  8  32 g
mS8 =
4  76 g
mS = 256 g
8

Resposta da questão 10:


[E]

[A] Incorreta. A transformação acontece com liberação de calor do meio externo ( ΔH  0 ) .


NH3 + HC → NH4C
− 46,1 kJ −92,3 kJ − 314,6 kJ
ΔH = Hprodutos − Hreagentes
ΔH =  − 314,6 kJ −  − 46,1 kJ + ( −92,3 kJ ) 
ΔH = −176,2 kJ  0 (reação exotérmica)

[B] Incorreta. A transformação é um exemplo de equilíbrio químico heterogêneo, pois o estado


de agregação do produto é diferente dos reagentes.

Página 45 de 105
Interbits – SuperPro ® Web

⎯⎯
NH3(g) + HC (g) ⎯→ NH4 C (s)

[C] Incorreta. A nuvem branca começa a se formar mais próxima do recipiente que contém
ácido muriático (solução aquosa de HC ).
NH3 = 1 14 + 3  1 = 17 
 36,5  17
HC = 1 1 + 1 35,5 = 36,5 

vHC MNH3 v 17
=  HC =
vNH3 MHC vNH3 36,5
vHC = 0,68  vNH3

De acordo com a lei de Graham, quanto maior a massa do gás, menor a sua velocidade e
vice-versa, consequentemente, como a massa do HC é maior, a nuvem branca começará
a se formar mais próxima do tambor de ácido muriático.

[D] Incorreta. O cloreto de amônio é um sal formado a partir de um ácido (HC ) forte e de uma
base fraca (NH4 OH ) , por isso, quando dissolvido em água, formará uma solução ácida.
NH4 C → NH+4 + C −

NH+4 + C − ⎯⎯
+ H2O ⎯→ H+ + C


+ NH3 + H2O

NH+4 ⎯
⎯⎯→ H+ + NH3

Meio
ácido

[E] Correta. De acordo com a lei de Graham, a nuvem branca começará a se formar em um
ponto mais próximo do recipiente que contém o ácido muriático, pois a massa do HC é maior,
do que a massa do NH3.

Resposta da questão 11:


a) Cálculo do valor numérico da ordem da reação:
Pmistura (início) = PA + PI
Pmistura (início) = 20 atm
4 mol de A : 1 mol de I (4 )  16 mol de A : 4 mol de I  16 atm (A) : 4 atm (I)
20 atm = 16 atm + 4 atm
Gás
inerte
2 A ( g) + I(g) ⎯⎯→ 3B ( g )
16 4 0 (atm; início)
− 2r (não reage) + 3r (atm; durante)
16 − 2r 4 3r (atm; total)
PA PB

Pmistura = 16 − 2r + 4 + 3r = 20 + r
Pmistura = 20 + r
r = 0, 1, 2, 3, 4, 5, 6

v (atm.min-1) PA = 16 – 2r (atm) P = 20 + r (atm) r PI (atm) T (min)


v = k(PA)a 16 – 0 = 16 20 0 4 0
1,96 16 – 2 = 14 21 1 4 0,89
1,44 16 – 4 = 12 22 2 4 2,08
1,00 16 – 6 = 10 23 3 4 3,75

Página 46 de 105
Interbits – SuperPro ® Web

0,64 16 – 8 = 8 24 4 4 6,25
0,36 16 – 10 = 6 25 5 4 10,42
0,16 16 – 12 = 4 26 6 4 18,75

t = 6,25 min  PA = 8 atm e v = 0,64 atm  min−1


t = 18,75 min  PA = 4 atm e v = 0,16 atm  min−1
v = k (PA )
a

( )
a
0,64 atm  min−1 = k ( 8 atm )  0,64 atm  min−1 k ( 8 atm )a k 23
a
0,64
 =  =
( )
−1
0,16 atm  min−1 = k ( 4 atm )  0,16 atm  min k ( 4 atm )
a a 0,16 a
k 22

22 = 2a  a = 2 (segunda ordem)

b) Cálculo do valor numérico da constante de velocidade com sua unidade de medida:


t = 18,75 min  PA = 4 atm e v = 0,16 atm  min−1
v = k (PA )  0,16 atm  min−1 = k ( 4 atm )2
2

0,16 atm  min−1


k=
16 atm2
k = 0,01 atm−1  min−1

c) Cálculo da composição no interior do reator no tempo 10,42 minutos em termos das


pressões parciais (em atm) de cada componente:
t = 10,42 min
r=5
PA = 16 − 2r = 16 − 2  5
PA = 6 atm
PI = 4 atm
PB = 3  r = 3  5
PB = 15 atm

d) Cálculo do valor numérico do tempo de meia vida da reação a partir de P A:

16 atm ⎯⎯⎯→
2 ( )
t 1
8 atm
0 min 6,25 min
t
( 12 ) = 6,25 min

Resposta da questão 12:


a) O gasoduto é o cátodo da célula eletroquímica, pois o aço precisa ser protegido, ou seja,
o ferro presente nesta liga metálica precisa receber elétrons do ânodo de sacrifício para não
sofrer corrosão.

b) Para ocorrer a proteção “catódica” do ferro presente no aço, o metal de sacrifício precisa
apresentar maior potencial de oxidação ou menor potencial de redução do que o ferro.
De acordo com a tabela fornecida no texto o único metal, na forma de cátion, que apresenta
menor potencial de redução do que o ferro (− 0,44 V) é o zinco (− 0,76 V).
Conclusão: o zinco deve ser utilizado para a proteção “catódica” do aço presente no
gasoduto.
O zinco será o polo negativo da célula eletroquímica, pois sofrerá oxidação (ânodo).

Página 47 de 105
Interbits – SuperPro ® Web

Fe2+ + 2 e− ⎯⎯→ Fe0 ε = − 0,44 V


2+ − 0
Zn + 2 e ⎯⎯→ Zn ε = − 0,76 V (inverter)
− 0,76 V  − 0,44 V
Ânodo −Oxidação
→ Zn2+ + 2 e−
( −) : Zn0 ⎯⎯⎯⎯⎯⎯⎯⎯
Cátodo −Re dução
( + ) : Fe2+ + 2 e− ⎯⎯⎯⎯⎯⎯⎯⎯
→ Fe0

c) Cálculo da massa, em kg, de etano existente em uma seção de 1 m3 do duto:


C2H6 (e tano) = 2  12 + 6  1 = 30; MC2H6 = 30 g  mol−1
P = 100 atm
V = 1 m3
T = 27 + 273 = 300 K
R = 8,2  10 −5 m3  atm  K −1  mol−1
% C2H6 (e tano) em mol = 4,1 % = 0,041
nC2H6 = 0,041 n
nC2H6 mC2H6
n=  n=
0,041 0,041 MC2H6
mC H
P V = nR T  P V = 2 6
R T
0,041 MC2H6
mC2H6
100 atm  1 m3 =  8,2  10 −5 m3  atm  K −1  mol−1  300 K
0,041 30 g  mol−1
100  1 0,041 30 g
mC2H6 = = 5  103 g
8,2  10−5  300
mC2H6 = 5 kg

Resposta da questão 13:


[D]

1NaHCO3 + 1HA → 1H2O + 1CO2 + 1NaA


gás
formado

VCO2 = Vútil − Vsolução


VCO2 = 74,9 mL − 50,0 mL = 24,9 mL

1mL = 10−6 m3  VCO2 = 24,9  10−6 m3

Cálculo da pressão PCO2 ( ) utilizando a equação fornecida na figura do texto:


t = 2 mm = 2  10−3 m
 = 0,050 V  (m  Pa)−1
U = 1,0 V
U =   t  PCO2

1,0 V = 0,050 V  (m  Pa)−1  2  10−3 m  PCO2


1,0 V
PCO2 = = 104 Pa
0,050 V  (m  Pa)−1  2  10−3 m

Página 48 de 105
Interbits – SuperPro ® Web

Utilizando a equação de Clapeyron, teremos:


1 Pa = 1 N  m−2  1 N = 1 Pa  m2
1 J = 1 N  m = 1 Pa  m3
R = 8,3 J  (mol  K)−1  R = 8,3 Pa  m3  (mol  K)−1
T = 300 K
PCO2  VCO2 = nCO2  R  T

104 Pa  24,9  10−6 m3 = nCO2  8,3 Pa  m3  (mol  K)−1  300 K

104 Pa  24,9  10−6 m3


nCO2 = = 10−4 mol
8,3 Pa  m3  (mol  K)−1  300 K

VHA (solução) = 50 mL = 50  10 −3 L

1NaHCO3 + 1HA ⎯⎯→1H2O + 1 CO2 + 1NaA


1mol 1mol
10−4 mol 10−4 mol
nHA 10−4 mol
HA  = =
VHA (solução) 50  10−3 L
HA  = 0,002 mol L

Resposta da questão 14:


[B]

Gráfico 1:
Material: CoBTTri
P = 1,0 atm
Quantidade adsorvida de O2: 5 mmol  L−1
Quantidade adsorvida de N2: 2 mmol  L−1

Página 49 de 105
Interbits – SuperPro ® Web

Gráfico 2:
Material: zeólito NaX
P = 1,0 atm
Quantidade adsorvida de O2: 0,125 mmol  L−1
Quantidade adsorvida de N2: 0,45 mmol  L−1

A partir da análise dos gráficos percebe-se que a separação do O2 e N2 pode ser feita
utilizando-se os dois materiais (CoBTTri e NaX).

Percebe-se pelo gráfico 1 que o oxigênio seria obtido na despressurização do sistema


referente ao complexo do metal cobalto denominado CoBTTri.

Resposta da questão 15:


[A]

H2 = 2  1 = 2
CH4 = 1 12 + 4  1 = 16
1
1min = h
60
km km
vH2 = 28 = 28 = 1680 km h
min  1 
h
 60 
 

A partir da Lei de Graham, vem:

Página 50 de 105
Interbits – SuperPro ® Web

v CH4 MH2 v CH4 2


=  =
vH2 MCH4 1680 km h 16
v CH4 2
=
1680 km h 4
1,4  1680 km h
v CH4 = = 588 km h
4
v CH4  600 km h

Resposta da questão 16:


[B]

Tinicial = 27 + 273 = 300 K


Tfinal = −196 + 273 = 77 K
Pinicial = 1 atm
Pinicial Pfinal
= (V constante)
Tinicial Tfinal
1 atm Pfinal
=
300K 77K
1 atm
Pfinal = 77K  = 0,2566666 atm
300K
Pfinal = 0,26 atm
1 atm 100%
0,26 atm p%
0,26 atm  100%
p% =
1 atm
p% = 26%

Resposta da questão 17:


[C]

Manhã fria:
T K = 273 + t C
T K = 273 + 10 = 283 K

Chegada ao destino:
T K = 273 + t C
T K = 273 + 30 = 303 K

Pneus da frente:
P P' 29 psi P'
=  =
T T' 283 K 303 K
29 psi
P' = 303 K  = 31psi (não precisa encher, pois atingiu o valor máximo da tabela)
283 K

Pneus de trás:

Página 51 de 105
Interbits – SuperPro ® Web

P P' 31psi P'


=  =
T T' 283 K 303 K
31psi
P' = 303 K  = 33,2 psi (precisa encher, pois não atingiu os 37 psi da tabela)
283 K

Resposta da questão 18:


a) Curva que representa o melhor material para se armazenar o hidrogênio: curva B.

Justificativa:
O material A possui menor capacidade de armazenamento de H2 , já os materiais B e C
apresentam capacidades próximas, logo o material A é descartado.

O material B apresenta uma menor distância entre as curvas de adsorção e dessorção, ou


seja, neste caso a reversibilidade do processo de armazenamento e liberação de H2 é mais
eficaz.

b) Uma possível desvantagem desta tecnologia alternativa estaria relacionada à massa


ocupada pelo Mg2Ni.

Justificativa:

Página 52 de 105
Interbits – SuperPro ® Web

dgasolina = 700 kg m−3


mgasolina = 24 kg
mgasolina mgasolina
dgasolina =  Vgasolina =
Vgasolina dgasolina
24 kg
Vgasolina = = 0,034 m3
700 kg m−3

mH2 = 8 kg
100 kg de Mg2Ni 3,6 kg de H2
mMg2Ni 8 kg de H2
100 kg  8 kg
mMg2Ni = = 222,2 kg
3,6 kg
dMg2Ni = 3400 k g  m−3
mMg2Ni mMg2Ni
dMg2Ni =  VMg2Ni =
VMg2Ni dMg2Ni
222,2 kg
VMg2Ni = = 0,065 m3
−3
3400 k g  m

Comparações:
mgasolina = 24 kg  24 kg
 Porcentagem ocupada pela gasolina (massa) =
mcarro médio = 1000 kg 1000 kg
Porcentagem ocupada pela gasolina (massa) = 2,4 %

Vgasolina = 0,034 m3  0,034 m3


 Porcentagem ocupada pela gasolina (volume) =
Vcarro médio = 6 m3  6 m3
Porcentagem ocupada pela gasolina (volume) = 0,57 %

mMg2Ni = 222,2 kg  222,2 kg


 Porcentagem ocupada pelo Mg2Ni (massa) =
mcarro médio = 1000 kg 1000 kg
Porcentagem ocupada pelo Mg2Ni (massa) = 22,2 %

VMg2Ni = 0,065 m3  0,065 m3


 Porcentagem ocupada pelo Mg2Ni (volume) =
Vcarro médio = 6 m3  6 m3
Porcentagem ocupada pelo Mg2Ni (volume) = 1,08 %

Porcentagem ocupada pelo Mg2Ni (massa) = 22,2 %


Porcentagem ocupada pela gasolina (massa) = 2,4 %
22,2 %
= 9,25
2,4 %

Conclusão: a porcentagem da massa do Mg2Ni é 9,25 vezes maior do que a da gasolina,


sendo isto uma enorme desvantagem.

Página 53 de 105
Interbits – SuperPro ® Web

Resposta da questão 19:


Supondo que as moléculas/partículas não possuam um volume igual a zero e possam ocupar
todo o recipiente, tem-se a seguinte correção de volume:

V = Videal + B
Videal = V − B
Pideal  Videal = n  R  T
Pideal  ( V − B ) = n  R  T (I)

Como o próximo às paredes do recipiente, as moléculas são atraídas ao centro com uma força
proporcional ao quadrado da concentração do gás, deve-se fazer uma correção da pressão,
pois esta será menor do que a ideal.

a : constante de proporcionalidade
n
Concentração =
V
2
n
P = Pideal − a  
V
2
n
Pideal = P + a   (II)
V

Substituindo (I) em (II), vem:


 n 
2
P + a     ( V − B ) = n  R  T
  V  

Levando-se em consideração o volume das próprias moléculas/partículas e fazendo-se a


correção para n mols (B = nb), obtém-se a equação de Van der Waals:
 n2 
P + a   ( V − nb ) = n  R  T
 V 2 

Resposta da questão 20:


a) De acordo com o texto, há a formação de monóxido de carbono na proporção molar de 1
para 8 em relação ao dióxido de carbono, então:
1n CO : 8n CO2 (proporção)  9 n mol de átomos de carbono
__ CnH2n + __ O2 → __ CO2 + __ CO + __ H2O
Balanceando :
9 CnH2n + 13n O2 → 8n CO2 + n CO + 9n H2O

A quantidade em mol de O2 que sobrou após a reação é igual à de carbono no início da


reação, ou seja, 9n mol. Então:
ntotal de O2 = 13n + 9n = 22n
nN2
= 3,76  nN2 = 3,76  nO2
nO2
nN2 = 3,76  22n = 83 mol

Cálculo da quantidade do número de mols dos gases resultantes da reação após a


combustão (considerando a remoção prévia da água):

Página 54 de 105
Interbits – SuperPro ® Web

ntotal = 9n (O2 em excesso) + 83n (N2 ) + 8n (CO2 ) + 1n (CO)


ntotal = 101n mol

Composição percentual aproximada:


nN2 83 n
= = 82%
ntotal 101n
nO2 9n
= = 9%
ntotal 101n
nCO2 8n
= = 8%
ntotal 101n
nCO 1n
= = 1%
ntotal 101n

b) Cálculo do percentual de ar em excesso na reação de combustão completa do alceno:


__ CnH2n + __ O2 → __ CO2 + __ H2O (combustão completa)
3
1 CnH2n + n O2 → n CO2 + n H2O
2
nO2 = 1,5 n
nO2 em excesso = nO2 (utilizado na combustão incompleta)
__ CnH2n + __ O2 → __ CO + __ H2O
1 CnH2n + n O2 → nCO + nH2O
nO2 em excesso = n mol
1,5n mol 100%
n mol p
n  100%
p= = 67%
1,5 n

Resposta da questão 21:


[A]

Variação máxima da oxigenoterapia: 10 L de O2 min.


Quando a pressão interna do gás oxigênio dentro do cilindro se igualar à pressão externa o
volume restante será de 50 L.

Página 55 de 105
Interbits – SuperPro ® Web

Vcilindro = 50 L
Pint erna = 200 atm
Pexterna = 1 atm
Vtotal (1 atm) = 10 m3 = 10000 L
Vutilizado de O2 = Vtotal (1 atm) − Vcilindro
Vutilizado de O2 = 10000 L − 50 L = 9950 L
10 L 1minuto
9950 L t
9950 L  1minuto
t= = 995 minutos
10 L
1hora 60 minutos
x 995 minutos
1hora  995 minutos
x= = 16,58 horas
60 minutos
x  17 horas

Resposta da questão 22:


(
a) MELHOR FONTE: Uthmaniyah 49,333 MJ m3 . )
PIOR FONTE: Uch (11,192 MJ m3 ) .

Justificativa: Comparação entre as médias ponderadas em MJ m3 das fontes listadas.

(
Maior média ponderada: Uthmaniyah 49,333 MJ m3 . )
(
Menor media ponderada: Uch 11,192 MJ m3 . )
Fontes
Laeq Uch Uthmaniyah
0,69  37 = 25,53 0,273  37 = 10,101 0,555  37 = 20,535
0,03  65 = 1,95 0,007  65 = 0,455 0,18  65 = 11,7
0,009  92 = 0,828 0,003  92 = 0,276 0,098  92 = 9,016
0,005  120 = 0,6 0,003  120 = 0,36 0,045  120 = 5,4
0,005  147 = 0,735 - 0,016  147 = 2,352
0,015  0 = 0 0,252  0 = 0 0,002  0 = 0
0,153  22 = 3,366 - 0,015  22 = 0,33
0,093  0 = 0 0,462  0 = 0 0,089  0 = 0
Soma Soma Soma
(Média ponderada) (Média ponderada) (Média ponderada)
33,009 MJ m3 11,192 MJ m3 49,333 MJ m3

b) MELHOR FONTE: Uch.


PIOR FONTE: Laeq.
Justificativa: dos componentes gasosos listados, o H2S é o mais poluente.
A fonte Uch não é composta por H2S e a fonte Laeq apresenta a maior quantidade de H2S em
sua composição (15,3 (V V %) ) .

Resposta da questão 23:


a) De acordo com o texto, a válvula de alívio de pressão que é acionada a 8,5 atm, sendo

Página 56 de 105
Interbits – SuperPro ® Web

que o reator contém uma mistura gasosa composta por quantidades iguais de um reagente
(A) e de uma substância inerte (B), a 10 C (TK = 273 + 10 = 283 K) e 2 atm.
nA = nB
T = 283 K
P = 2 atm  p A + pB = 2 atm
p A = 2 − pB
p A pB ( 2 − pB ) pB
=  =
P P 2 2
pB = 1 atm
p A = 1 atm

A temperatura é elevada para 293 C (TK = 293 + 273 = 566 K), então:
pA pA f 1 atm pA f
=  =
TA T ' A 283 K 566 K
p A = 2 atm
f

pB pBf 1 atm pBf


=  =
TA T ' A 283 K 566 K
pBf = 2 atm

A partir da equação fornecida no texto, vem:


566 K
2 A (g) ⎯⎯⎯⎯
→ 3 C(g) + 4 D(g) + 1E(g)
2 0 0 0 (início; atm)
−2p + 3p + 4p +p (durante; atm)
( 2 − 2p ) +3p +4p +p (final; atm)
pA pC pD pE

pBf = 2 atm
Ptotal = 8,5 atm
Ptotal = pBf + p A + pC + pD + pE
8,5 = 2 + ( 2 − 2p ) + 3p + 4p + p
8,5 = 4 + 6p
p = 0,75 atm
Consumo de A = −2 p  Consumo de A = −2  0,75 atm
Δp A = −1,5 atm (consumo de A)

De acordo com o texto, a velocidade de consumo de A nessa temperatura é dada por


v A = −0,25  (PA )0 (em atm  h−1 ), onde PA corresponde à pressão parcial da substância A.

v A = −0,25  (PA )0
v consumo de A = −0,25
Δp A
= −0,25
Δt
−1,5
= −0,25
Δt
Δt = 6 h

A válvula de alívio é acionada após 6 horas.

Página 57 de 105
Interbits – SuperPro ® Web

b) Pressões parciais de cada espécie (A, B, C, D e E) presente no reator no momento do


acionamento da válvula de alívio: 0,5 atm, 2 atm, 2,25 atm, 3 atm e 0,75 atm,
respectivamente.
pB = 2 atm
f
566 K
2 A (g) ⎯⎯⎯⎯
→ 3 C(g) + 4 D(g) + 1E(g)
2 0 0 0 (início; atm)
−1,5 + 3  0,75 + 4  0,75 + 0,75 (durante; atm)
0,5 atm 2,25 atm 3 atm 0,75 atm (final; atm)
pA pC pD pE

c) Para que a válvula de alívio não seja acionada, o reagente A tem que ser totalmente
consumido (p A = 0 atm; 100% de ren dim ento) e a pressão total não poderá ultrapassar a
soma p A + pC + pD + pE = 8,5 atm.
pA + pB + pC + pD + pE = ( 0,5 + 2 + 2,25 + 3 + 0,75 ) atm
pA + pB + pC + pD + pE = 8,5 atm
566 K
2 A (g) ⎯⎯⎯⎯
→ 3 C(g) + 4 D(g) + 1E(g)
x 0 0 0 (início; atm)
−x + 1,5x + 2x + 0,5x (durante; atm)
0 +1,5x +2x + 0,5x (final; atm)
pC pD pE
pB = x
Ptotal = 8,5 atm
Ptotal = x + 0 + 1,5x + 2x + 0,5x
8,5 = 5x
x = 1,7 atm
A 10 C : Pmistura = x = 1,7 atm
A 293 C : Pmistura = 2x = 2  1,7 atm = 3,4 atm

Supondo que o valor do volume do reator seja igual a Vreator :


R = 8,21 10 −2 atm  L  K −1  mol−1
Pmistura  Vreator = nmistura  R  T
1,7  Vreator = nmistura  8,21 10−2  283
1,7
nmistura =  Vreator
8,21 10−2  283
nmistura  ( 0,07  Vreator ) mol

Resposta da questão 24:


[B]

Cálculo do número de mols de gás hidrogênio:

Página 58 de 105
Interbits – SuperPro ® Web

T = 27 + 273 = 300 K
P = 1,00 atm
V = 480 mL = 480  10 −3 L
R = 0,08 atm  L  mol−1  K −1
P V
P  V = nH2  R  T  nH2 =
RT
1,00 atm  480  10 −3 L
nH2 =
0,08 atm  L  mol−1  K −1  300 K
nH2 = 0,02 mol

Cálculo da massa molar (MM ) do metal:


1M + 2HC ⎯⎯→ 1H2 + 1MC 2
MM 1 mol
2,38 g 0,02 mol
2,38 g  1 mol
MM =
0,02 mol
MM = 119 g
Sn = 119  M = Sn (es tanho)

Resposta da questão 25:


[D]

A partir das informações fornecidas, vem:


P = 1 atm
R = 0,082 atm  L  mol−1  K −1
T(Kelvin) = 27 + 273 = 300 K
P  Vmolar = n  R  T
1 Vmolar = 1 0,082  300
Vmolar = 24,6 L
NaHCO3 = 23 + 1 + 12 + 3  16 = 84

[I] Incorreta. A equação química balanceada que descreve corretamente a reação citada (ácido
clorídrico reage com bicarbonato de sódio e produz gás carbônico) é:
1HC + 1NaHCO3 → 1H2O + 1 CO2 + 1NaC

[II] Correta. Para a formação de 6,15 L de CO2, foram consumidos 21 g de bicarbonato de


sódio.
1HC + 1NaHCO3 → 1H2O + 1 CO2 + 1NaC
84 g 24,6 L
m 6,15 L
84 g  6,15 L
m= = 21 g
24,6 L

[III] Incorreta. O valor dos números de oxidação (Nox) se mantém, ou seja, não é uma reação
de oxidorredução.
1 H C + 1 Na H C O O O → 1 H H O + C O O + 1Na C
+1 −1 +1 +1 +4 −2 −2 −2 +1 +1 −2 +4 −2 −2 +1 −1

Página 59 de 105
Interbits – SuperPro ® Web

[IV] Correta. Se todo esse ácido clorídrico fosse consumido numa reação completa com
bicarbonato de sódio suficiente, o volume de gás carbônico produzido seria de 9,84 L.
1HC + 1NaHCO3 → 1H2O + 1 CO2 + 1NaC
1mol 24,6 L
0,4 mol VCO2
0,4 mol  24,6 L
VCO2 = = 9,84 L
1mol

Resposta da questão 26:


[D]

A partir da equação de estado para um gás ideal, calcula-se o número total de mols no reator.
P = 32,8 atm
V = 200 L
T = 400 K
R = 8,21 10 −2 atm  L  mol1  K −1  8,2  10 −2 atm  L  mol1  K −1
P V = nR T
32,8 atm  200 L = n  8,2  10 −2 atm  L  mol1  K −1  400 K
32,8 atm  200 L
n= = 200 mols
−2
8,2  10 atm  L  mol1  K −1  400 K

No início: pCO = 3  pNO2 .


A partir da análise do equilíbrio, vem:
1NO2 + 1 CO ⎯⎯⎯→ 1 CO2 + 1NO
⎯⎯⎯
pNO 3  pNO 0 0 (início)
2 2
−p −p +p +p (durante)
(pNO 2 ) (3  pNO − p )
−p 2
+p +p (final)

Pfinal = ( pNO − p ) + ( 3  pNO )


−p +p+p
2 2
32,8 = 4  pNO
2
32,8
pNO2 = = 8,2 atm
4
pCO = 3  pNO
2
pCO = 3  8,2 atm = 24,6 atm

 mi 
 
= i
pi ni pi M
= 
P n 32,8 200
200
mi =  pi  Mi
32,8
NO2 = 14 + 2  16 = 46; CO = 12 + 14 = 28
200
mNO2 =  8,2  46 = 2300 g = 2,3 kg
32,8
200
mNO2 =  24,6 atm  28 = 4200 g = 4,2 kg
32,8

Resposta da questão 27:

Página 60 de 105
Interbits – SuperPro ® Web

[A]

9
mH2O(v) =  4,5 mg = 0,405  10 −3 g
100
MH O = ( 2  1 + 16 ) g mol = 18 g  mol−1
2

V = 0,1mL = 0,1 10 −3 L
R = 0,082 L  atm  mol−1  K −1
T = 177 + 273 = 450 K
P V = nR T
m
P V = R T
M
0,405  10−3 g
Pfinal  0,1 10−3 L =  0,082 L  atm  mol−1  K −1  450 K
18 g  mol−1
Pfinal = 8,30025 atm  8,3 atm

A associação correta entre região do milho e Pfinal é dada por:


A = endosperma e Pfinal = 8,3 atm.

Resposta da questão 28:


[D]

Página 61 de 105
Interbits – SuperPro ® Web

Me tano ( CH4 = 16 )  80% em volume


E tano ( C2H6 = 30 )  10% em volume
Pr opano ( C3H8 = 44 )  5% em volume
Nitrogênio (N2 = 28 )  5% em volume
mtotal = 100 g
Mmédia = 0,80  16 + 0,10  30 + 0,05  44 + 0,05  28 = 19,4 g  mol −1
mtotal 100 g
ntotal = = = 5,1546 mol
Mmédia 19,4 g  mol−1
Vgás ngás
=
Vtotal ntotal
ngás
% Volume =  ngás = ( % Volume )  ntotal
ntotal
n
(C 1H4 ) = 0,80  ntotal  nC = 1  0,80  5,1546 mol = 4,12368 mol

n
(C 2 H6 ) = 0,10  ntotal  nC = 2  0,10  5,1546 mol = 1,03092 mol

n
(C H ) = 0,05  ntotal  nC = 3  0,05  5,1546 mol = 0, 77319 mol
3 8

n(N ) = 0,05  ntotal  nC = 0  0,05  5,1546 mol = 0,00 mol


2

nC = ( 4,12368 + 1,03092 + 0,77319 ) mol = 5,92779 mol


Butadieno ( C4H6 ) = 54
4 mol C 54 g de butadieno
5,92779 mol C m( C
4H6 )
5,92779 mol C  54 g
m( C = = 80,025165 g
4H6 ) 4 mol C
m( C = 80 g
4H6 )

Resposta da questão 29:


[C]

De acordo com a figura, em relação às quantidades de gases dissolvidos no magma, é correto


afirmar que as concentrações de SO2 são maiores que as de HF e de HC nos três vulcões,
pois as relações molares estão indicadas acima de 1 no gráfico.

n SO2 n SO2
1 e 1
n HF n HC

Página 62 de 105
Interbits – SuperPro ® Web

n SO2 n SO2
Para o vulcão Erta Ale e estão entre 1 e 10. Logo, as concentrações de HF e
n HF n HC
HC são aproximadamente iguais.

n SO2 n SO2
Para o vulcão Kilauea e estão entre 10 e 100. Logo, as concentrações de HF
n HF n HC
e HC são aproximadamente iguais.

Página 63 de 105
Interbits – SuperPro ® Web

n SO2 n SO2
Para o vulcão Colima está entre 10 e 100, sendo que está entre 1 e 10, ou
n HF n HC
seja, não coincidem. Logo as concentrações de HF e HC não são aproximadamente iguais.

Resposta da questão 30:


[B]

Página 64 de 105
Interbits – SuperPro ® Web

m
P V = nR T  P V = R T
M
m PM P M
=  d=
V RT RT
MX = 32 g  mol−1; dX = 2,0 g  L−1; dY = 30 g  L−1; R = cte
Para X :
P  32 g  mol−1 2,0 g  L−1 P
2,0 g  L−1 =  =
RT − 1 RT
32 g  mol
Para Y :
P  MY g  mol−1 3,0 g  L−1 P
3,0 g  L−1 =  =
RT −1 T
MY g  mol R
P P
=
RT RT
2,0 g  L−1 3,0 g  L−1
=
32 g  mol−1 MY g  mol−1
32 g  mol−1  3,0 g  L−1
MY =
2,0 g  L−1
MY = 48 g mol

Resposta da questão 31:


a) Cálculo da pressão inicial do vapor de água utilizado nesse lançamento:
V : volume inicial do cilindro
c : comprimento inicial do cilindro  c = 3 m
d : diâmetro do cilindro
2
d
V = π  c
2
V ' : volume final do cilindro (após a exp ansão do gás)
c ' : comprimento final do cilindro (após a exp ansão do gás)  f = 90 m
d : diâmetro do cilindro
2
d
V' = π  c'
2
P : pressão inicial do cilindro  P = ?
P' : pressão final do cilindro  Pf = 1 atm
Numa transformação isotérmica, o produto P×V é constante
(temperatura constante de 500 K):
P  V = P' V '
2 2
d d
P  π     c = P' π     c '
2 2
P  c = P' c '
P  0,6 m = 1 atm  90 m  P = 30 atm

b) Relação entre o número de mols de N2 e H2O :

Página 65 de 105
Interbits – SuperPro ® Web

m =m = m = constante
(N2 ) (H2O)

m
(N2 ) m
n
(N2 ) 28 28 18
= = = = 0,642857  0,64
n m m 28
(H2O) (H2O)
18 18
n = 0,64  n
(N2 ) (H2O)

Então, P = 0,64  P
(N2 ) (H2O)

P = 0,64  30
(N2 )

P = 19,2 atm
(N2 )

19,2 atm  30 atm

A pressão necessária para o lançamento a 90 m não seria atingida e o lançamento não seria
bem-sucedido.

Resposta da questão 32:


[C]

V1  V2  P1  P2 (de acordo com a figura)


α -Transformação
P1  V1 isotérmica (T =T ) P2  V2
⎯⎯⎯⎯⎯⎯⎯⎯
1 2 →
T1 T2
P1  V1 = P2  V2 (hipérbole equilátera)

V2  V3  T2  T3 (de acordo com a figura)


β-Transformação
P2  V2 isobáricaca (P =P ) P3  V3
⎯⎯⎯⎯⎯⎯⎯⎯→
2 3
T2 T3
V2 V3
P = cons tan te; =
T2 T3

Página 66 de 105
Interbits – SuperPro ® Web

Conclusão:

Resposta da questão 33:


[D]

Página 67 de 105
Interbits – SuperPro ® Web

T = 27 C + 273 = 300 K; V = 240 L; R = 8,21 10 −2 atm  L  mol−1  K −1


1
pCO = pH  pH = 3pCO
2 3 2 2 2

1
nCO2 = nH2  nH2 = 3nCO2
3
ntotal = nH2 + nCO2 = 3nCO2 + nCO2 = 4nCO2
P  V = ntotal  R  T
0,82  240 = 4nCO2  8,21 10 −2  300
0,82  240
nCO2 = = 1,9976 mol  2 mol
4  8,21 10−2  300
mCO = 2  44 g
2

mCO2 = 88 g
nH2 = 3nCO2  nH2 = 3  2 = 6 mol
mH2 = 6  2 g

mH2 = 12 g

Resposta da questão 34:


a) Aplicando a lei de Hess para as informações fornecidas na tabela, vem:
A + B ⎯⎯→ C Qv = 400 kJ mol (manter)
D+ 1
2B ⎯⎯→ C Q v = 300 kJ mol (inverter)

A + B ⎯⎯→ C Q1 = 400 kJ mol


C ⎯⎯→ D + 1 B
2 Q2 = −300 kJ mol
Global
A+ 1 B
2 ⎯⎯⎯⎯
→D Qtotal = ( +400 − 300) kJ
Qtotal = +100 kJ
Global
A + 1
2B ⎯⎯⎯⎯
→D Qtotal = +100 kJ
1 mol 100 kJ liberados
0,1 mol E
E = 100  0,1kJ
E = 10 kJ liberados (10.000 J)

b) Determinação da temperatura final do banho térmico:


Qtotal = 10.000 J; n = 0,4 mol; C V,m = 83,33 J K –1mol –1; Ti = 300 K.
Qtotal = n  C V,m  ΔT
Qtotal = n  C V,m  ( Tf − Ti )
10.000 J = 0,4 mol  83,33 J K –1mol –1  ( Tf − 300 K )
10.000 J
Tf − 300 K =
0,4 mol  83,33 J K –1mol–1
Tf = 600,012 K
Tf = 600 K

c) Considerando que o gás não é ideal e aplicando a equação de Van der Waals para gases
reais, vem:

Página 68 de 105
Interbits – SuperPro ® Web

n = 0,4 mol; a = 62,5 L2atm mol–2 ; b = 0,4 L mol –1;


V = 10 L; Ti = 300 K; R = 8,21 10 −2 atm L K −1 mol−1.
 n2a 
 Pinicial + 2  ( V − nb ) = nRTi
 V 

 (0,4 )2  62,5 
 Pinicial +  (10 − (0,4 )  0,4 ) = (0,4 )  8,21 10 −2  300
 2 
 (10 ) 
(Pinicial + 0,1)  9,84 = 9,852
9,84  Pinicial = 9,852 − 0,984
9,852 − 0,984
Pinicial = = 0,9012 atm
9,84
Pinicial = 0,9 atm

Analogamente,
n = 0,4 mol; a = 62,5 L2atm mol–2 ; b = 0,4 L mol –1;
V = 10 L; Tf = 600 K; R = 8,21 10 −2 atm L K −1 mol−1.
 n2a 
 Pfinal + 2  ( V − nb ) = nRTf
 V 

 (0,4)2  62,5 
 Pfinal +  (10 − (0,4)  0,4 ) = (0,4)  8,21 10 −2  600
 2 
 (10) 
(Pfinal + 0,1)  9,84 = 19,704
9,84  Pfinal = 19,704 − 0,984
19,704 − 0,984
Pfinal = = 1,9024 atm
9,84
Pfinal = 1,9 atm

Resposta da questão 35:


[C]

Transformação isocórica significa a volume constante.


Pinicial  V Pfinal  V
=
Tinicial Tfinal
Pinicial Pfinal
=
Tinicial Tfinal
Tinicial = 30 C + 273 = 303 K
Pinicial = 2 atm
Tfinal = 303 K + 150 K = 453 K
Pinicial Pfinal
=
Tinicial Tfinal
2 atm P
= final
303 K 453 K
2 atm
Pfinal =  453 K
303 K
Pfinal = 2,99 atm (aproximadamente 3 atm)

Página 69 de 105
Interbits – SuperPro ® Web

Resposta da questão 36:


a) A partir das informações do enunciado, vem:
m 13,1 g
n Xe = = = 0,1 mol
M 131 g  mol−1

1Xe(g) + 2F2(g) ⎯⎯→ 1XeF4(g)


1 mol 2 mol 1 mol
0,1 mol 0,2 mol (0,1 mol + n F2(excesso) )
6 atm 0,1 mol + nF2(excesso)
2,4 atm nF2(excesso)
6  nF2(excesso) = 2,4  (0,1 mol + nF2(excesso) )
6  nF2(excesso) = 0,24 + 2,4  nF2(excesso)
3,6  nF2(excesso) = 0,24  nF2(excesso) = 0,0666666 mol
mF2(excesso) = 0,0666666  38 g = 2,5333308 g
mF2(excesso) = 2,53 g (massa que não reagiu)

b) 1 Xe(g) + 2F2(g) ⎯⎯→ 1 XeF4(g) .


A

c) Reação da platina sólida com XeF4 :


A X
Pt + XeF4 ⎯⎯→ Xe + PtF4
195 g 271 g
19,5 g m PtF4
m PtF4 = 27,1 g

Resposta da questão 37:


[D]

Hélio: 30.000 cm3 (30 L) a 760 cmHg (7.600 mmHg = 10 atm) e 27 C (300 K)
R = 8,21 10−2 atm  L  mol−1  K −1
pHe  V
pHe  V = nHe  R  T  nHe =
RT
10  30
nHe = = 12,18 mol
8,21 10−2  300

Página 70 de 105
Interbits – SuperPro ® Web

Monóxido de carbono: 250 L a 1.140 mmHg (1,5 atm) e −23 C (250 K)


pCO  V
pCO  V = nCO  R  T  nCO =
RT
1,5  250
nCO = = 18,27 mol
8,21 10−2  250

Monóxido de nitrogênio: 2 m3 (2.000 L) a 0,273 atm e 0 C (273 K)


pNO  V
pNO  V = nNO  R  T  nNO =
RT
0,273  2.000
nNO = = 24,36 mol
8,21 10−2  273
ntotal = nHe + nCO + nNO = 12,18 mol + 18,27 mol + 24,36 mol = 54,81 mol
pHe n p 12,18
= He  He =
Ptotal ntotal 4,5 54,81
pHe = 1,05 atm  1,0 atm

Resposta da questão 38:


a) Equação química que justifica a efervescência devido à formação de gás carbônico
(CO2 ):
Tem-se uma mistura sólida que contém os íons dihidrogenofosfato, H2 PO 4− , e
hidrogenocarbonato, HCO3− .
H2 PO−4 + HCO3− ⎯⎯→ HPO24− + H2CO3
H+ HPO24− H2O+CO2

H2 PO−4 + HCO3− ⎯⎯→ HPO24− + H2O + CO2 


Efervescência

b) 18 g desse fermento químico devem liberar, no mínimo, 1,45  10−3 m3 de gases a 298 K e
93.000 Pa, (dado: R = 8,3 Pa m3 mol−1 K −1), então:
MCO2 = 44g  mol−1
m
P V = R  T
M
mCO2
93.000 Pa,1,45  10 −3 m3 =  8,3 Pa m3 mol−1 K −1  298 K
44 g  m ol−1
mCO2  2,4 g

NaHCO3 = 84
H2 PO4− + HCO3− ⎯⎯→ HPO24− + H2O + CO2 
Efervescência

H2 PO4− + NaHCO3 ⎯⎯→ HPO24− +


+ Na + H2O + CO2 
Efervescência
84 g 44 g
mNaHCO3 2,4 g
mNaHCO  4,58 g
3

Página 71 de 105
Interbits – SuperPro ® Web

Resposta da questão 39:


A proposta da questão é hipotética, por isso desconsidera fatores biológicos. Sendo assim, a
partir dos dados fornecidos, e supondo que o ar aspirado seja composto por,
aproximadamente, x% de oxigênio e y% de nitrogênio, podemos fazer os cálculos solicitados.

Na superfície:
Psup erfície = 1 atm = 10 mH2O
Pembaixo da água = 200 mH2 O = 20  Psup erfície = 20  1 = 20 atm
Ptotal = Psup erfície + Pembaixo da água = 1 + 20 = 21 atm

21 atm 100%
pO 2 x%
pO2 = 0,21 x atm

21 atm 100%
pN2 y%
pO2 = 0,21 y atm

Para x = 20% de oxigênio e y = 20% de nitrogênio, vem:


Psup erfície = 1 atm = 10 mH2O
Pembaixo da água = 200 mH2 O = 20  Psup erfície = 20  1 = 20 atm
Ptotal = Psup erfície + Pembaixo da água = 1 + 20 = 21 atm

21 atm 100%
pO 2 20%
pO2 = 4,2 atm

21 atm 100%
pN2 80%
pO2 = 16,8 atm

Resposta da questão 40:


[B]

Em cada abertura da garrafa ocorre o escape de gás carbônico (CO2 ) e a pressão interna
diminui e se iguala à externa, ou seja, 1 atm.
Quando a garrafa é novamente fechada, a pressão interna aumenta com a liberação de gás
carbônico, porém não atinge o valor inicial de 3,5 atm.
Quanto mais gás escapa, menor a pressão interna após cada abertura. Este comportamento é
mostrado no gráfico [B].

Página 72 de 105
Interbits – SuperPro ® Web

Resposta da questão 41:


a) A temperatura dos pneus, recolhidos na fuselagem, era −13 C ( −13 + 273 = 260 K)
durante o voo. Próximo ao pouso, a temperatura desses pneus passou a ser
27 C (27 + 273 = 300 K) e a pressão de 30 atm, mas seu volume interno não varia, ou seja,
trata-se de uma transformação isovolumétrica.

Ppneus recolhidos Ppneus pouso


=
Tpneus recolhidos Tpneus pouso
Ppneus recolhidos 30 atm
=
260 K 300 K
30 atm  260 K
Ppneus recolhidos =
300 K
Ppneus recolhidos = 26 atm

b) Cálculo do volume interno desse mesmo pneu, em litros, dado que foram utilizados 14 kg
(14  103 g) de N2 ara enchê-lo:

mN2 14  103 g
nN2 = = = 500 mol
MN2 28 g  mol−1
nN2 = 500 mol; T = 300 K; P = 30 atm; R = 0,082 L  atm  K -1  mol-1
P V = nR T
30 atm  V = 500 mol  0,082 L  atm  K -1  mol-1  300 K
V = 410 L

c) Fórmula estrutural do monômero do poli-isopreno, ou seja, do isopreno:

Então,

Ou seja,

Página 73 de 105
Interbits – SuperPro ® Web

Resposta da questão 42:


[D]

3 3
pNO =  pH2  nNO =  nH2
5 5
mNO mNO mH2 mH2
nNO = = mol; nH2 = = mol
MNO 30 MH2 2
mNO = 9  mH2
mNO + mH2 = 20 g
9  mH2 + mH2 = 20 g  mH2 = 2 g
2g
p%(H2 ) = = 0,1 = 10%
20 g

Resposta da questão 43:


[C]

Análise do ciclo termodinâmico ideal de três etapas:

[I] Aquecimento isocórico (combustão).


Num aquecimento isocórico (volume constante), a pressão aumenta e o volume permanece
constante, ou seja, V1 = V2 , então teríamos o trecho representado abaixo.

[II] Expansão adiabática (liberação de gases).


Numa expansão quase-estática adiabática P  V γ = constante.
Onde,
Cp
γ= 1
Cv
Cp : capacidade calorífica à pressão cons tan te.
Cv : capacidade calorífica a volume cons tan te.

Como γ  1, no plano cartesiano a curva da transformação adiabática é mais inclinada do


que as isotermas.

Página 74 de 105
Interbits – SuperPro ® Web

P2  V2 γ = Pk  Vk γ
V2 γ Pk
=
γ P2
Vk
γ
 V2  Pk
  =
 k
V P2
V  P P V 
γ  1   2   k ou k   2 
 k
V P2 P2  Vk 
V 
Pk  P2   2 
 Vk 

[III] Compressão isobárica (rejeição de calor a pressão atmosférica).


Numa compressão isobárica a pressão permanece constante.

Resposta da questão 44:


a) De acordo com o texto e o diagrama mostrado na figura abaixo, do ponto (1) ao (2) a
transformação é isocórica (volume constante; V1 = V2 ). Neste caso, a temperatura máxima
é de 1.200 K, pois acima deste valor o material do motor começa a sofrer desgaste:

Página 75 de 105
Interbits – SuperPro ® Web

Patm = 1 atm
T1 = 300 K
T2 = 1.200 K (temperatura suportável pelo material)
Patm P2
=
T1 T2
1 atm P2
=
300 K 1.200 K
P2 = 4 atm

b) Numa transformação isocórica (a volume constante), como não há alteração de volume, o


trabalho realizado é nulo. A quantidade de calor (Q) necessária para variar a temperatura
( T) de um sistema de massa m é dada por: Q = m  c  ΔT, onde c é o calor específico a
volume constante.
m
Como n =  m = n  M, vem:
M
ΔT = Tfinal − Tinicial = 1.200 − 300 = 900 K
n = 0,2 mol
CV = 25 J  K −1  mol−1
Q = n  M  c V  ΔT  Q = n  C V  ΔT
Calor
molar
a volume
cons tan te
(CV )

Q = 0,2 mol  25 J  K −1  mol−1  900 K


Q = 4.500 J = 4,5 kJ

CH4(g) + 2 O2(g) → CO2(g) + 2 H2O(g) Q V = 45 kJ  g−1


1g 45 kJ
mCH4 4,5 kJ
mCH = 0,1 g
4

Resposta da questão 45:


[E]

P V = nR T
2,46  1 = nH2  0,082  (27 + 273)
nH2 = 0,1 mol
Fe(s) + 2HC (aq) → H2 (g) + FeC 2 (aq)
56 g 1 mol
p  11,2 g 0,1 mol
p = 0,50 = 50 % (teor de ferro)

Resposta da questão 46:


[C]

A transformação isotérmica é aquela que ocorre à temperatura constante.

Conhecendo-se a curva, no plano PV, de um processo de expansão nós podemos calcular o


trabalho realizado.

Página 76 de 105
Interbits – SuperPro ® Web

Se o gás for ideal e a temperatura constante podemos usar a equação de estado PV = nRT,
portanto:
nRT nRT
P=  PdV = dV
V V

Ou seja:

O trabalho (W ) realizado pelo gás, quando ele sofre expansão isotérmica de Vi até Vf é dado
por:
W = P  ΔV
ou
Vf Vf dV V
W= V PdV = nRT V
i i V
= nRT n f
Vi

Na figura anterior o trabalho (W) é representado pela área hachurada.

A energia interna (U) depende da temperatura, como na transformação isotérmica a


temperatura não se modifica o calor removido é igual ao trabalho realizado pelo gás.
Q= U +W Q=W
zero
ou
dQ = dW

Análise das alternativas:


[A] (Incorreta) O trabalho realizado pelo gás é diferente nos dois processos de expansão.

I. Expansão em uma etapa, contra a pressão externa constante de 1 atm, levando o volume
final do recipiente a 10 L. Temperatura constante.
WI = P  ΔV
WI = 1 atm  (10 L − 1 L) = 9 atm.L

II. Expansão em duas etapas: na primeira, o gás expande contra a pressão externa
constante de 5 atm até atingir um volume de 2 L; na segunda etapa, o gás expande
contra uma pressão constante de 1 atm atingindo o volume final de 10 L.
WII = P  ΔV + P ' ΔV '
WII = 5 atm  (2 L − 1 L) + 1 atm  (10 L − 2 L)
WII = 13 atm  L

[B] (Incorreta) O trabalho realizado no primeiro processo é igual a 9 atm  L e o trabalho


realizado no segundo processo é igual a 13 atm  L.

Página 77 de 105
Interbits – SuperPro ® Web

[C] (Correta) A variação da energia interna do gás e igual em ambos os processos.


Como a variação da energia interna do gás é igual em ambos os processos e vale zero, pois
a temperatura, em ambos os casos, é constante.

[D] (Incorreta) A variação da energia interna do gás nos dois casos é igual a zero, pois a
temperatura é constante.

[E] (Incorreta) O calor trocado pelo gás é igual ao trabalho nos dois casos, portanto é diferente.

Resposta da questão 47:


a) De acordo com o gráfico a condição em que homens e mulheres teriam mesma
capacidade orgânica máxima de absorver, transportar e utilizar o oxigênio do ar atmosférico,
por massa corporal, para a produção da energia via aeróbia seria o cruzamento das curvas
HS (homens sedentários) e MA (mulheres ativas), pois neste ponto ocorre coincidência. Este
ponto está dentro da faixa etária 5.

b) Analisando o gráfico, vem:

Página 78 de 105
Interbits – SuperPro ® Web

Tempo = 60 min
Por quilograma de massa corporal em 1 min a capacidade é de 32 mL, então :
V60 minutos = 32 mL  60 = 1920 mL de O2
Massa da mulher = 58 kg
V ' = 1920 mL  58 = 111.360 mL de O2
1 mol 25.000 mL
n 111.360 mL
n = 4,4544 mol
MO2 = 32 g / mol
mO2 = 4,4544  32 g = 142,5408 g
mO2  142,5 g

Resposta da questão 48:


a) Primeiro ensaio: os balões foram inicialmente evacuados e, logo a seguir, com a torneira
fechada, foram introduzidos 0,30 g de benzeno e 20,0 g de tolueno em “A” e “B”.
Dados do cabeçalho da prova:
Pr essão de vapor do benzeno puro a 298 K = 100,00 mmHg
Pr essão de vapor do tolueno puro a 298 K = 30,00 mmHg

Balão A (benzeno; C6H6 )


m 0,30 g
nbenzeno = = = 0,0038461 mol
M 78 g  mol−1
R = 62,3 mmHg  Lmol−1  K −1; V = 894 mL = 0,894 L; T = 298 K
P V = nR T
P  0,894 = 0,0038461 62,3  298
P = 79,870666 mmHg  79,87 mmHg
79,87 mmHg  100,0 mmHg
Conclusão: o benzeno evapora totalmente.
Pressão no balão A no primeiro ensaio = 79,87 mmHg

Página 79 de 105
Interbits – SuperPro ® Web

Balão B (tolueno; C7H8 )


m 20,0 g
ntolueno = = = 0,2173913 mol
M 92 g  mol−1
R = 62,3 mmHg  Lmol−1  K −1; V = 894 mL = 0,894 L; T = 298 K
P V = nR T
P  0,894 = 0,2173913  62,3  298
P = 4.514,4922 mmHg  4,514,49 mmHg
4,514,49 mmHg  30,0 mmHg
Conclusão: o tolueno não evapora totalmente, teremos uma fase líquida e uma gasosa.
Pressão no balão B no primeiro ensaio = 30,0 mmHg

b) Lei de Raoult: a pressão máxima de vapor de uma solução (Psolução) será igual ao produto da
fração molar do solvente (Xsolvente) com a pressão máxima de vapor do solvente puro (Psolvente
puro).

Ou seja,
Psolução = Xsolvente  Psolvente puro

No momento em que o equilíbrio líquido-vapor é atingido a pressão interna é de


76,2 mmHg.
Pvapor do benzeno = 100,0 mmHg
Pvapor do tolueno = 30,0 mmHg
Pequilíbrio líquido− vapor = 76,2 mmHg
Psolução = Xbenzeno  Pvapor do benzeno + X tolueno  Pvapor do tolueno
Pequilíbrio líquido− vapor = Xbenzeno  Pvapor do benzeno + Xtolueno  Pvapor do tolueno
76,2 = Xbenzeno  100,0 + X tolueno  0,30

1 = Xbenzeno + X tolueno
76,2 = Xbenzeno  100,0 + (1 − Xbenzeno )  30,0
76,2 = Xbenzeno  100,0 + 30,0 − 30,0 Xbenzeno
70Xbenzeno = 76,2 − 30,0
Xbenzeno = 0, 66

Como 1 = Xbenzeno + X tolueno ,


Xtolueno = 1 − 0,66 = 0,34  X tolueno = 0,34
Pfase gasosa do benzeno Pfase gasosa do tolueno
= 0,66; = 0,34
Pvapor do benzeno Pvapor do tolueno
Pfase gasosa do benzeno
= 0,66  Pfase gasosa do benzeno = 66 mmHg
100,0
Pfase gasosa do tolueno
= 0,34  Pfase gasosa do benzeno = 10,2 mmHg
30,0
Pfase gasosa do benzeno 66
Xbenzeno no equilíbrio líquido-vapor = = = 0,8661417  0,87
Pequilíbrio líquido− vapor 76,2
Pfase gasosa do tolueno 10,2
Xtolueno no equilíbrio líquido-vapor = = = 0,1338582  0,13
Pequilíbrio líquido− vapor 76,2

Resposta da questão 49:


[E]

Página 80 de 105
Interbits – SuperPro ® Web

A energia cinética média de um gás ideal é dada por:


3
Ec =  n  R  T
2

Onde,
n : número de mols do gás
R : constante universal dos gases ideais
T : temperatura na escala Kelvin

Supondo-se o número de mols constante, vem:


3
Ec =  n  R  T
2
cons tan te
Ec = a  T
Ec : y
T: x
y = ax (equação de uma reta que passa na origem do sistema)

Resposta da questão 50:


a) mgasolina = ?
dgasolina = 0,740 g  cm−3 = 740 g / L
1 L de gasolina 740 g
45 L de gasolina mgasolina
mgasolina = 33.300 g = 33,3 kg

b) VH2 ( ) =?

Página 81 de 105
Interbits – SuperPro ® Web

dH2 ( ) = 0,071 g  cm−3 = 71 g / L


mH2 ( )
dH2 ( ) =
VH2 ( )
mH2 ( )
VH2 ( ) =  mH2 ( ) = (71VH2 ( ) ) g
71
PC : poder calorífico
Energia E
PC = =
massa m
PCH2 = 3  PCgasolina
EH2 Egasolina
= 3
mH2 mgasolina
EH2 Egasolina
= 3
71VH2 ( ) 33.300
EH2 = Egasolina = E (mesma quantidade de energia liberada)
E E
= 3
71VH2 ( ) 33.300
VH2 ( ) = 156,3380282 L  156,34 L

c) mH2 ( = (71VH2 ( ) ) g
)
g
mH2 ( ) = 71  156,3380282 L
L
mH2 ( ) = 11.100 g
MH2 = 2  1,01 = 2,02 g / mol
P = 1 bar
1 atm = 1,01325 bar
R = 0,082  1,01325 bar  L  mol−1  K −1
T = 25 C = 273 = 298 K
mH2
P  VH2 = R T
MH2
11.100
1 VH2 =  0,082  1,01325  298
2,02
VH2 = 136.056,2003 L

VH2  1,36  105 L

Resposta da questão 51:


a) Para calcular a massa de ar presente na cápsula utiliza-se a equação de estado de um
gás (Clapeyron):

Página 82 de 105
Interbits – SuperPro ® Web

P = 100.000 Pa
V = 1,0 cm3 = 10−3 L = 10−6 m3
M = 29 g  mol−1
R = 8,3 Pa  m3  K −1
T = 25 C + 273 = 298 K
m
P V = R T
M
mar
100.000 Pa  10−6 m3 =  8,3 Pa  m3  K −1  298 K
−1
29 g  mol
mar = 0,0011724 g = 1,1724 mg
mDBO 1,5 mg
= = 1,2794268
mar 1,1724 mg
mDBO
 1,28
mar

b) Cálculo do decaimento para o DBO:

t 1 = 12 h
2
12 h
1,5 mg ⎯⎯⎯→ 0,75 mg
Massa de DBO após 12 h = 0,75 mg.

Cálculo do decaimento para o ecstasy:

12
t 1 = 1,5 h  = 8 períodos de meia − vida
2 1,5
1,5 h 1,5 h 1,5 h 1,5 h 1,5 h
10 mg ⎯⎯⎯→ 5 mg ⎯⎯⎯→ 2,5 mg ⎯⎯⎯→ 1,25 mg ⎯⎯⎯→ 0,625 mg ⎯⎯⎯→
1,5 h 1,5 h 1,5 h
0,3125 mg ⎯⎯⎯→ 0,15625 mg ⎯⎯⎯→ 0,078125 mg ⎯⎯⎯→ 0,0390625 mg
Massa de ecstasy após 12 h = 0,0390625 mg  0,039 mg

O indivíduo que teria maior massa do princípio ativo da droga após 12 horas seria aquele
que ingeriu DBO, pois 0,75 mg  0,039 mg.

Resposta da questão 52:


[D]

Página 83 de 105
Interbits – SuperPro ® Web

Uma mistura gasosa ideal não reagente, formada por 10 g de gás hidrogênio, 10 g de gás
hélio e 70 g de gás nitrogênio encontra-se acondicionada em um balão de volume igual a 5 L,
sob temperatura de 27  C, então a partir dos dados, vem:

Na mistura, tem-se:
m 10 g 
nH2 = = = 5 mols 
M 2 g  mol−1 

m 10 g 
nHe = = = 2,5 mols  ntotal = 5 + 2,5 + 2,5 = 10 mols
M 4 g  mol − 1

m 70 g 
nN2 = = = 2,5 mols 
M 28 g  mol−1 

A pressão parcial nas condições citadas no texto é diretamente proporcional ao número de


mols parcial. Conclusão:
pHe n 2,5
= He =  pHe = 0,25  Pmistura
Pmistura ntotal 10
pN2 nN2 2,5
= =  pN2 = 0,25  Pmistura
Pmistura ntotal 10
pH2 nH2 5
= =  pH2 = 0,50  Pmistura
Pmistura ntotal 10
1
pHe = pN2 = pH
2 2

T = 27 + 273 = 300 K
Pmistura  V = ntotal  R  T
Pmistura  5 = 10  0,082  300
Pmistura = 49,2 atm

pHe nHe 2,5


XHe = = = = 0,25 = 25 %
Pmistura ntotal 10

VH2 nH2 5
= =
V ntotal 10
VH2 5
=  VH2 = 2,5 L
5 10

Resposta da questão 53:


[D]

A parir dos valores fornecidos podemos calcular a massa molar (M) do gás.

Página 84 de 105
Interbits – SuperPro ® Web

R = 62,4 mmHg  L  mol−1K −1


m = 4,4 g
V = 3,1 L
T = 10 C + 273 = 283 K
P = 566 mmHg
m
P V = R T
M
4,4 g
566 mmHg  3,1 L =  62,4 mmHg  L  mol−1K −1  283 K
M
M = 44,28387 g  mol−1

A razão entre as densidades (neste caso massas específicas) dos gases é igual à razão entre
as massas molares, então:
Mgás = 44,28387 g  mol−1

MH2 = 2,02 g  mol−1


dgás Mgás
=
dH2 MH2
dgás 44,28387 g  mol−1
= = 21,922707
dH2 2,02 g  mol−1
dgás
 22
dH2

Resposta da questão 54:


[C]

Contribuíram de forma direta para o desenvolvimento do conceito de pressão atmosférica


Galileu Galilei, que desenvolveu a medição de temperatura e Evangelista Torricelli que mediu a
pressão atmosférica em nível do mar através do uso do mercúrio (experiência de Torricelli).

Resposta da questão 55:


a) Não concordo. O ar que respiramos é composto por, aproximadamente, 20% de gás
oxigênio e 80% de gás nitrogênio. O gás que vazou é 100% puro, ou seja, deve ser inalado
em quantidades controladas.
A inalação de gás oxigênio puro pode levar ao desequilíbrio do metabolismo.

b) A estratégia utilizada para que eles não tenham tido êxito foi o fato dos motoristas terem
tentado tirar os carros com os motores ligados.
O gás vazou e como o oxigênio (32g / mol) é mais denso do que o ar (  29g / mol) ficou
próximo ao solo favorecendo a combustão e o incêndio do primeiro carro que estava ligado (já
que o gás oxigênio acelera vigorosamente a combustão). Se os outros motoristas tentassem
ligar seus carros iniciariam novas combustões.

Resposta da questão 56:


[A]

[A] O desodorante aerossol, possui em sua composição os gases: Butano, Isobutano e


Propano.
[B] O extintor de incêndio possui o dióxido de carbono.
[C] Gás de cozinha: mistura dos gases propano e butano.
[D] O gás natural veicular ou gás natural é composto em sua maior parte pelo gás metano.
[E] O gás refrigerante de geladeira: é o gás tetrafluoretano, usado atualmente, pois é isento de
cloro e causando menos dano à camada de ozônio.

Página 85 de 105
Interbits – SuperPro ® Web

Resposta da questão 57:


[D]

Análise das afirmações:

[I] Correta. Se o gás for resfriado contra pressão externa constante, o sistema contrai-se
(volume e temperatura serão diretamente proporcionais).
A partir da equação geral dos gases, vem :
Pantes  Vantes Pdepois  Vdepois
=
Tantes Tdepois
Pantes = Pdepois = P (cons tan te)

P  Vantes P  Vdepois
=
Tantes Tdepois
Vantes Vdepois
=
Tantes Tdepois

[II] Errada. A elevação da temperatura não está diretamente ligada à variação da posição do
pistão (volume).

[III] Correta. Se o sistema for aquecido a volume constante, a velocidade média das moléculas
aumenta, independentemente da natureza do gás, pois a energia cinética é diretamente
proporcional à temperatura.

[IV] Errada. De acordo com a Lei de Graham, teremos:


v Xe MAr
=
v Ar MXe
v Xe 40
=
v Ar 131
40
v Xe =  v Ar
131
Conclusão : v Xe  v Ar .

Resposta da questão 58:


[E]

Teremos:

PTOTAL = Par sec o + Pacetona


760 = Par sec o + 180
Par sec o = 580 mmHg

O tambor foi danificado e seu volume interno diminuiu para 80% do volume inicial, sem que
tenha havido vazamento e a temperatura foi mantida constante, então:

Pinicial  Vinicial = Pfinal  Vfinal


Pinicial = Pacetona
Vfinal = 0,80Vinicial
580  Vinicial = Pfinal  0,80 Vinicial
Pfinal = 725 mmHg

Página 86 de 105
Interbits – SuperPro ® Web

PTOTAL APÓS A QUEDA = Pfinal + Pacetona


PTOTAL APÓS A QUEDA = 725 + 180 = 905 mmHg

Resposta da questão 59:


a) Um gás possui uma taxa de efusão que corresponde a 25,0% da taxa do gás hidrogênio,
ou seja: v X = 0,25vH2 .

A partir da lei da efusão gasosa, teremos:

vX MH2
=
vH2 MX
0,25vH2 2 2
=  0,25 =
vH2 MX MX
2
(0,25)2 =
MX
2
MX = = 32 g.mol−1
(0,25)2

T= 39,5°C + 273=312,5 K; R = 0,082 atm.L.mol-1.K −1; MX = 32 g.mol−1


P  V = nX  R  T
mX
1 1 =  0,082  312,5
32
m X = 1,24878 g = 1,249 g

b) A partir da lei de Raoult, teremos:

ΔP
= x soluto
p0
nsoluto
x soluto =
nsoluto + nsolvente

Numa solução muito diluída nsolvente é muito maior do que nsoluto :


nsoluto + nsolvente  nsolvente

Página 87 de 105
Interbits – SuperPro ® Web

msoluto
nsoluto Msoluto M  msoluto
x soluto  = = solvente
nsolvente msolvente msolvente  msoluto
Msolvente
Então :
ΔP ΔP Msolvente  msoluto
= x soluto  =
p0 p0 msolvente  msoluto
ou
ΔP  Msolvente   1000  msoluto 
=  
p0  1000   msolvente  Msoluto 
ΔP  Macetona   1000  msulfanilamida 
=  
400  1000   macetona  Msulfanilamida 
ΔP  58   1000  1,249 
= 
400  1000   100  172 
ΔP = 1,68469 mmHg
400 − p = 1,68469
p = 398,315 mmHg

Resposta da questão 60:


[B]

Um volume V1 de oxigênio e um volume V2 de ácido sulfídrico, ambos nas mesmas condições


de temperatura e pressão, são misturados, então:

2H2S(g) + 3O2 (g) → 2SO2 (g) + 2H2O( )


2 vol 3 vol 2 vol
V1 (Total de O2 ) + V2 (H2S) = 24
VO (reage) + VO (excesso)
2 2

 VO2 (reage) + VO2 (excesso) + V2 (H2S) = 24



 VSO2 + VO2 (excesso) = 10
3 vol + VO2 (excesso) + 2 vol = 24 14
  vol =
 2 vol + VO2 (excesso) = 10 3
V2 (H2S) = 3 vol
14
V2 (H2S) = 3  = 9,33 L
3

Resposta da questão 61:


a) Teremos:
Carbonato de sódio + ácido sulfúrico, então:
Na2CO3(s) + H2SO4(aq) → H2O( ) + CO2(g) + Na2SO4(aq)

b) O experimento descrito foi repetido utilizando-se carbonato de potássio em lugar de


carbonato de sódio, então: K 2CO3(s) + H2SO4(aq) → H2O( ) + CO2(g) + K 2SO4(aq) As massas
molares do Na2 CO3 e K 2CO3 são diferentes. Como as massas não foram fornecidas,
devemos lembrar que, na tabela periódica, a massa atômica do potássio é maior do que a do
sódio, pois o potássio, na família IA, está abaixo do sódio.
Concluí-se que a massa molar do K 2CO3 (MM' ) é maior do que Na2CO3 (MM) .

Página 88 de 105
Interbits – SuperPro ® Web

Para uma mesma massa m dos dois compostos, teremos:


MM'  MM
m m
nNa2CO3 = ; nK 2CO3 =
MM MM'
nNa2CO3  nK 2CO3

Como a quantidade de gás carbônico formada varia de acordo com o número de mols de
Na2 CO3 e K 2CO3 , concluí-se que o número de mols de gás carbônico ( CO2 ) formado na
reação com carbonato de sódio ( Na2 CO3 ) é maior do que na reação com carbonato de
potássio ( K 2CO3 ).

nNa2CO3(s) + _ H2SO4(aq) → _ H2O( ) + nCO2(g) + _ Na2SO4(aq)


n'K 2CO3(s) + _ H2SO4(aq) → _ H2 O( ) + n'CO2(g) + _ K 2 SO4(aq)

A altura atingida pelo êmbolo não será a mesma, pois dependerá da quantidade de gás
carbônico liberada.

c) O volume do cilindro pode ser dado por: Volume = área da base ( π  (raio)2 ) x altura (x)
V = π  r 2  x . Como o volume é dado por volume é dado por V = nRT/P, teremos:
nR T m
V= ; n=
P MM
n  R  T m RT
π  r2  x =  π  r2  x = 
P MM P
mR T
x=
MM  P  π  r 2

Resposta da questão 62:


[C]

Sabemos, a partir da Lei de Henry, que a concentração molar de uma solução é diretamente
proporcional (K = constante de proporcionalidade) a sua pressão parcial (p) na fase gasosa,
então:

[N2 ] = K  pN2  5,85  10−4 mol / L = K  0,78 atm

K = 7,5  10−4 mol  L−1  atm−1

A variação da concentração molar também é diretamente proporcional à variação da pressão


parcial e a constante de proporcionalidade é a mesma encontrada anteriormente, logo:

[N2 ] = K  pN2

[N2 ] = 7,5  10−4 mol  L−1  atm−1  (5,00 atm − 0,78 atm)
[N2 ] = 31,65  10−4 mol  L−1
nN2
[N2 ] =
V
nN2
31,65  10−4 mol  L−1 =
6L
nN2 = 189,9  10−4 mol  1,9  10 −2 mol

Resposta da questão 63:

Página 89 de 105
Interbits – SuperPro ® Web

a) Teremos:
m
P V = R T
M
m PM PM
= d=
V RT RT

Como P e R são cons tan tes :


PM
d=
RT

P  16 P
dMe tano = = 16
RT RT
P  28,8 P
dMédia do ar = = 28,8
RT RT
P
dMédia do ar 28,8 RT
= = 1,8 dMe tano
dMe tano P
16
RT
dMe tano  dMédia do ar

O metano ascende na atmosfera, pois sua densidade é menor do que a densidade média do
ar.

b) Não seria possível obter a mistura com a composição acima mencionada pela simples
mistura desses gases. O ar tem, aproximadamente, 21 % em volume de gás oxigênio. Se o
metano fosse misturado ao oxigênio, este apresentaria uma porcentagem menor que 21 % na
mistura.

Resposta da questão 64:


Teremos:

PCNTP = 1 atm
1 = X AB2 + X AB
1 = 0,1+ X AB
X AB = 0,9
p(gás)
X(fração molar) =
P(total)
p AB2 = 0,1 1 atm
p AB = 0,9  1 atm
AB2 ( g ) + A (s) 2 AB ( g )
0,1 1 atm 0,9  1 atm (equilíbrio)
2 2
p AB (0,9)
KP = = = 8,1
1 0,1
p AB2

Para um novo valor de pressão, vem:

Página 90 de 105
Interbits – SuperPro ® Web

KP = 8,1
AB2 ( g ) + A ( s ) 2 AB ( g )
0,2  P 0,8P (equilíbrio)
2 2
p AB (0,8 P)
KP = = = 3,2  P
1 0,2 P
p AB2
KP = 3,2
8,1 = 3,2  P
P = 2,53 atm

Resposta da questão 65:


[A]

De acordo com a Lei de Graham:

v = velocidade
M = massa molar
v1 M2
=
v2 M1

HC = 36,5 
M  MNH
NH3 = 17  HC 3

vHC MNH3 v 17
=  HC =
vNH3 MHC vNH3 36,5
vHC  vNH3

O anel se forma mais próximo do HC , pois a velocidade de efusão da amônia gasosa é maior.

O anel branco formado é o NH4 C sólido, resultado da reação química entre HC e NH3
gasosos.

Resposta da questão 66:


a) De acordo com a figura, o anel de NH4C se forma a 6,0 cm da extremidade do algodão
com HC e a 9,0 cm da extremidade do algodão com NH3 . Quanto maior a distância, maior
a velocidade do gás no tubo, concluí-se que o NH3 é o gás que apresenta maior velocidade
de difusão.
b) Quanto maior a temperatura, maior a velocidade de difusão das moléculas e a velocidade da
reação. Consequentemente o anel de será formado num tempo menor do que 10 s.
c) Caso o algodão embebido de solução aquosa de NH3 (g) seja colocado no tubo um pouco
antes do algodão que libera HC (g) (e não simultaneamente) o anel de NH4C será formado
a uma distância maior da extremidade do algodão embebido com NH3 dando a impressão de
que a velocidade de difusão do HC é menor do que a verdadeira. De acordo com a expressão
matemática fornecida, quanto menor a velocidade de difusão, maior a massa molar.
Consequentemente, a massa molar do HC parecerá maior do que a verdadeira.

Resposta da questão 67:


a) Na figura, uma solução concentrada de HC , contida em A, é gotejada sobre zinco sólido
em B. A reação que ocorre em B é a seguinte: 2HC (conc) + Zn(s) → H2 (g) + ZnC 2 (aq) .
A substância presente inicialmente em C é um líquido marrom-avermelhado (substância
simples, cujo elemento pertence à família 17) à temperatura ambiente, ou seja, trata-se do

Página 91 de 105
Interbits – SuperPro ® Web

bromo líquido (Br2 ) que passa para a forma de vapor. A reação que ocorre em C na
presença de gás hidrogênio é a seguinte: H2 (g) + Br2 (v) → 2HBr(g) .

b) A pressão não deve exceder 2 atm (C possui volume de 1,0 L, é mantido a 100°C), então:
P V = nR T
2  1 = n  0,082  (100 + 273)
n = 0,065565 mol

Inicialmente existe 0,05 mol de Br2 (v), logo:


nmáximo = n − 0,05 mol = 0,065565 mol − 0,05 mol = 0,015565 mol
nmáximo = 0,015565 mol = 1,56  10−2 mol

Resposta da questão 68:


a) Equação química balanceada da combustão completa do acetileno com oxigênio puro:
2C2H2 + 5O2 → 4CO2 + 2H2O

b) Como as condições de pressão, temperatura e volume são iguais, os dois cilindros contêm o
mesmo número de mols de moléculas. Para um mol de moléculas, teremos:
Massa molar média do ar = 28,9 g/mol.
Massa molar do oxigênio = 32 g/mol.

O cilindro de oxigênio puro contém uma massa maior.

c) Como a porcentagem de oxigênio é maior no oxigênio puro do que no ar, a energia liberada
será maior na queima do acetileno na presença do oxigênio puro.
Na queima do acetileno com gás oxigênio puro, o calor liberado será absorvido pela água e
pelo gás carbônico.
Já no caso da queima do acetileno na presença de ar, o calor liberado será absorvido
pela água, pelo gás carbônico e pelo gás nitrogênio (maior número de substâncias), logo a
temperatura do sistema será menor.

Resposta da questão 69:


[E]

α é a pressão, β é o volume, δ é a temperatura e o processo é um aquecimento isobárico (a


pressão constante).

Resposta da questão 70:


[A]

Página 92 de 105
Interbits – SuperPro ® Web

Como a massa do sistema é constante, teremos:


Pinicial  Vinicial Pfinal  Vfinal
=
Tinicial Tfinal

I. Processo isobárico de T0 até T0/2.


P0  V0 PI  VI
=
T0 TI
P  V0 P  VI V U
=  VI = 0  UI = 0
T0 T0 2 2
2

II. Processo isobárico de V0 até 2V0.


P0  V0 PII  VII
=
T0 TII
P  V0 P  2V0
=  TII = 2T0  UII = 2U0
T0 TII

III. Processo isocórico de P0 até P0/2.


P0  V0 PIII  VIII
=
T0 TIII
P0
P0  V V T U
= 2  TIII = 0  UIII = 0
T0 TIII 2 2

IV. Processo isocórico de T0 até 2T0.


P0  V0 PIV  VIV
=
T0 TIV
P0  V PIV  V
=  PIV = 2P0
T0 2T0
TIV = 2T0  UIV = 2U0

V. Processo isotérmico de P0 até P0/2.


P0  V0 PV  VV
=
T0 TV
P0
P0  V0  VV
= 2  VV = 2V0
T T
TV = T0  UV = U0

VI. Processo isotérmico de V0 até V0/2.


P0  V0 PVI  VVI
=
T0 TVI
V0
P0  V0 PVI  2
=  PVI = 2P0
T T
TVI = T0  UVI = U0

Numa transformação isotérmica UV = UVI.

Resposta da questão 71:

Página 93 de 105
Interbits – SuperPro ® Web

[D]

Um recipiente de paredes rígidas, contendo apenas ar, aberto para a atmosfera está submetido
à pressão constante e tem volume fixo.

Logo,

Pi  Vi = ni  R  Ti (a 27 C ou 300 K)
Pf  Vf = nf  R  Tf (a 127 C ou 400 K)

Então,

Pi  Vi = ni  R  Ti (I)
Pf  Vf = nf  R  Tf (II)
Pi  Vi = Pf  Vf = P  V
Dividindo (I) por (II) :
P V n  R  300 n 300 n
= i  f =  f = 0,75
P  V nf  R  400 n i 400 ni
mf  mf
nf =
Mar  Mar
 = 0,75
mi  mi
ni =
Mar  Mar
mf
= 0,75  mf = 0,75mi
mi
msaiu = 1,00mi − 0,75mi = 0,25mi
Conclusão : a porcentatem mássica de ar que saiu é de 25%.

Resposta da questão 72:


[D]

A partir da equação de Clapeyron (equação do estado de um gás), vem:


m
PV= RT
M
7,4
P  0,8 =  0,08  (37 + 273)
74
P = 3,1 atm

Resposta da questão 73:


[E]

Análise das afirmações:

I. Afirmação incorreta. De acordo com a hipótese de Avogadro (gases ideais) como os dois
cilindros se encontram na mesma temperatura e pressão e ocupam o mesmo volume,
possuem a mesma quantidade de moléculas.

II. Afirmação incorreta. A velocidade das moléculas no cilindro C1 é menor do que no cilindro
C2, pois a massa molar do O2 é maior do que a do Ne. As velocidades das partículas que
formam a mistura gasosa são inversamente proporcionais à raiz quadrada das respectivas
massas molares ou moleculares.
v O2 MNe
= .
vNe MO2

Página 94 de 105
Interbits – SuperPro ® Web

III. Afirmação correta. A densidade do gás no cilindro C1 é maior que a densidade do gás no
PM
cilindro C2. A densidade pode ser obtida pela seguinte relação: d = .
RT
Como a pressão e a temperatura são as mesmas nos dois cilindros, concluímos que a
densidade do gás oxigênio é maior do que a do gás neônio.

IV. Afirmação incorreta. A distribuição de velocidades das moléculas contidas no cilindro C1 e


menor que a das contidas no cilindro C2.

Resposta da questão 74:


a) A pressão parcial do CO2 pode ser calculada por:
PCO2 0,037
= x CO2  PCO2 =  1 = 3,7  10−4 atm
P 100

De acordo com a lei de Henry, vem:


[CO2(aq) ] = K  PCO2
[CO2(aq) ] = 3,4  10 −2  3,7  10 −4
[CO2(aq) ] = 12,58  10 −6
[CO2(aq) ] = 1,258  10 −5 = 1,3  10 −5 mol.L−1

Em miligramas, teremos:
c = 1,3  10 −5  44 g.L−1
c = 1,3  10 −5  44  103 mg.L−1
c = 5,72  10 −1mg.L−1

b) O equilíbrio pode ser representado por:

CO2(aq) + H2 O( ) H+(aq) + HCO3 −(aq)


1,3  10 −5 M x x

[H+ ][HCO3 − ]
K=
[CO2 ]
x2
4,4  10 −7 =
1,3  10 −5
x = 2,39  10 −6 M  [HCO3 − ] = 2,39  10 −6 M

Resposta da questão 75:


[A]

Equação da combustão completa do propano:


C3H8 + 5O2 → 3CO2 + 4H2O

5 mols O2 100 % de ar
nO2 80 % de ar
n = 4 mols de O2
4 mols de O2 1 parte
nN2 4 partes
nN2 = 16 mols de N2

Conclui-se que a combustão será incompleta, já que são utilizados 4 mols de gás oxigênio:

Página 95 de 105
Interbits – SuperPro ® Web

C3H8 + 4O2 → 1CO2 + 2CO + 4H2O

Levando em consideração o gás nitrogênio, vem:


C3H8 + 4O2 + 16N2 → 1CO2 + 2CO + 4H2O + 16N2

nTotal = nCO2 + nCO + nH2O + nN2


nTotal = 1 + 2 + 4 + 16 = 23 mols
23 mols 100%
1 mol p%CO
2
p%CO = 4,3478% = 4,35%
2

Resposta da questão 76:


[A]

O sistema registra um aumento de 5% na pressão e de 15 °C na temperatura:

Pinicial = Pi
Aumento de 5%  Pfinal = Pi + 0,05Pi = 1,05Pi
Tfinal = Tf = Ti + 15
Transformação ivolumétrica ou isocórica :
Pi Pf
=
Ti Tf
Pi 1,05 Pi
=
Ti Ti + 15
Ti + 15 = 1,05Ti
0,05Ti = 15
Ti = 300 K
TK = 273,15 + TC
300 = 273,15 + TC
TC = 26,85 C

A temperatura é inferior a 30°C.

Resposta da questão 77:


[E]

De acordo com a equação de estado de um gás ideal, vem:

Página 96 de 105
Interbits – SuperPro ® Web

P V = nR T
n = constan te
P V = nR  T
constan te
P V = kT
V
P =k
T
V
Pe são inversamente proporcionais.
T
V
Então para Pmáxima , tem que ser mínima.
T
V
P   = k
T

Isto ocorre no gráfico da alternativa [E]:

Resposta da questão 78:


T = 273 + 100 = 373 K
ntotal = nY + nN2
Ptotal  V = ntotal  R  T
5  2 = ntotal  0,082  373
ntotal = 0,327 mol

ntotal = nY + nN2

m Y m N2
ntotal = +
MY MN
2
8,4 6,4
0,327 = +
MY 28
MY = 85,37 g / mol  86 g / mol

X é um brometo de alquila:

CnH2n+ 2Br = 86
12n + 2n + 2 + 80 = 86
n=6
CnH2n+ 2  C6H14 (Y)

Página 97 de 105
Interbits – SuperPro ® Web

X é opticamente ativo, ou seja, possui carbono assimétrico:

Y é deriva da substituição do bromo pelo radical etil:

Resposta da questão 79:


Uma massa de óxido ferroso é aquecida a 1273 K e, em seguida, exposta a uma mistura
gasosa de monóxido de carbono e hidrogênio:

FeO + CO → CO2 + Fe H1


FeO + H2 → CO2 + H2O H2

FeO → Fe Ha = +265 kJ


H2 → H2O Hb = −250 kJ
CO → CO2 Hc = −282kJ
Ha + Hc = +265 − 282 = −17 kJ
1 mol CO − 17 kJ
nCO x
x = −17nCO
Ha + Hb = +265 − 250 = +15 kJ
1 mol H2 + 15 kJ
nH2 y
y = +15nCO

Perda de calor total = x + y = −4,2 kJ

−17nCO + 15nH = −4,2(nCO + nH )


2 2
−17nCO + 15nH2 = −4,2nCO − 4,2nH2
12,8nCO = 19,2nH2
nCO 19,2 p
= = 1,5  CO = 1,5
nH 12,8 pH
2 2

Página 98 de 105
Interbits – SuperPro ® Web

Resposta da questão 80:


A transformação isovolumétrica de um gás triatômico hipotético A 3 em outro diatômico A 2
envolve a liberação de 54 kJ/mol de A 3 :

3
1A 3 (g) → A 2 (g) H = −54 kJ
2

Teremos:

3
1A 3 (g) → A 2 (g) H = −54 kJ
2
3
x x
2
nA 3 = x
nA 2 = 1,5x

Q = nA 2  Cv  T

54x kJ = 54  103 x J
54  103 x = 1,5x  30  T
T = 1200 o C

Aplicando a equação de estado de um gás (Clapeyron):

T = 1200 o C  T = 1500 − 300 = 1200 oC


Pinicial  V = x  R  300
Pfinal  V = 1,5x  R  1500
Pinicial  V x  R  300
=
Pfinal  V 1,5 x  R  1500
Pinicial 300 P 1,5  1500
=  final = = 7,5
Pfinal 1,5  1500 Pinicial 300
Pfinal
= 7,5  Pfinal = 7,5  Pinicial
Pinicial
Pinicial 100%
7,5  Pinicial p
p = 750%
Aumento de pressão = 750% − 100% = 650%

Resposta da questão 81:


[C]

Solubilidade do CO2 no refrigerante a 5 C e sob 1 atm de CO2 = 3,0 g L


MCO2 = 44 g mol
1mol 44 g de CO2
nCO2 3g
1mol  3 g  3 
nCO2 = =  mol
44 g  44 

Página 99 de 105
Interbits – SuperPro ® Web

Volume molar dos gases a 1 atm e 30 C = 25 L mol


25 L 1mol
 3 
VCO2  44  mol
 
 3 
 44  mol  25 L
VCO2 =  = 1,7045454 L
1mol
VCO2  1,7 L

Resposta da questão 82:


ANULADA (sem resposta)

Análise das afirmações:


Alternativas [A] e [B]:
Assumindo a alternativa [A] como verdadeira, entra-se em contradição com a alternativa [B],
pois em [A] diz-se que a parte do hidróxido que está dissolvida encontra-se dissociada
ionicamente e, em [B], diz-se que nem toda quantidade dissolvida está dissociada ionicamente.
Uma das duas alternativas teria que ser falsa.

A alternativa [C] é verdadeira.


A condutividade elétrica da mistura do frasco A é a maior porque se trata de uma solução 1
molar (0,1 mol / 0,1 L) de eletrólito forte (100% de dissolução e 100% de dissociação iônica).

A alternativa [D] é falsa.


Não se pode afirmar que os três solutos são bases fortes, tendo em vista a baixa
condutibilidade elétrica nos frascos B e C comparativamente ao frasco A. Além disso, o
hidróxido de cobre II e o hidróxido de níquel II são compostos de coordenação que sofrem
baixa ionização.

A alternativa [E] é verdadeira.


Soluções muito diluídas com igual concentração normal destes 3 hidróxidos deveriam
apresentar condutividades elétricas semelhantes.

Resposta da questão 83:


Zinco metálico reage com solução de ácido clorídrico da seguinte maneira:

Zn(s) + 2HC (aq) → H2 (g) + ZnC 2 (aq)


1 mol 2 mols 1 mol 1 mol
m
nZn = nH2 =
M
130
nZn = = 2 mols
65,0
nH2 = 2 mols

Página 100 de 105


Interbits – SuperPro ® Web

Para o I2 (g) :
PI2 = 3,38 atm; T = 227 + 273 = 500 K; R = 0,082 atm.L.mol−1.K −1; V = 50 L
PI2  V = nI2  R  T
3,28  50 = nI2  0,082  500
nI2 = 4 mols
Então,
H2 (g) + I2 (g) 2HI(g)
2 mols 4 mols 0 (início)
−x −x + 2x (durante − estequiometria)
(2 − x) (4 − x) 2x (equilíbrio)
ntotal = (2 − x) + (4 − x) + 2x = 6 mols
Pfinal  V = ntotal  R  T
Pfinal  50 = 6  0,082  500
Pfinal = 4,92 atm

Página 101 de 105


Interbits – SuperPro ® Web

Resumo das questões selecionadas nesta atividade

Data de elaboração: 10/06/2023 às 16:54


Nome do arquivo: Gases

Legenda:
Q/Prova = número da questão na prova
Q/DB = número da questão no banco de dados do SuperPro®

Q/prova Q/DB Grau/Dif. Matéria Fonte Tipo

1 ............. 198225 ..... Elevada ......... Química ......... Enem PPL/2020 ................... Múltipla escolha

2 ............. 189830 ..... Elevada ......... Química ......... Unicamp/2020 ...................... Múltipla escolha

3 ............. 221453 ..... Elevada ......... Química ......... Fuvest/2023 ......................... Analítica

4 ............. 217130 ..... Elevada ......... Química ......... Unicamp/2023 ...................... Múltipla escolha

5 ............. 220630 ..... Elevada ......... Química ......... Ime/2023 .............................. Analítica

6 ............. 220531 ..... Elevada ......... Química ......... Ime/2023 .............................. Múltipla escolha

7 ............. 222353 ..... Elevada ......... Química ......... Enem PPL/2022 ................... Múltipla escolha

8 ............. 205226 ..... Elevada ......... Química ......... Ita/2022 ................................ Analítica

9 ............. 205223 ..... Elevada ......... Química ......... Ita/2022 ................................ Analítica

10 ........... 204952 ..... Elevada ......... Química ......... Upe-ssa 2/2022.................... Múltipla escolha

11 ........... 205221 ..... Elevada ......... Química ......... Ita/2022 ................................ Analítica

12 ........... 205274 ..... Elevada ......... Química ......... Fuvest/2022 ......................... Analítica

13 ........... 203134 ..... Elevada ......... Química ......... Ime/2022 .............................. Múltipla escolha

14 ........... 203711 ..... Elevada ......... Química ......... Unicamp/2022 ...................... Múltipla escolha

15 ........... 196123 ..... Elevada ......... Química ......... Espcex (Aman)/2021 ........... Múltipla escolha

16 ........... 198861 ..... Elevada ......... Química ......... Fgv/2021 .............................. Múltipla escolha

17 ........... 196796 ..... Elevada ......... Química ......... Unicamp/2021 ...................... Múltipla escolha

18 ........... 197600 ..... Elevada ......... Química ......... Unicamp/2021 ...................... Analítica

19 ........... 196033 ..... Elevada ......... Química ......... Ime/2021 .............................. Analítica

20 ........... 198486 ..... Elevada ......... Química ......... Ita/2021 ................................ Analítica

21 ........... 196738 ..... Elevada ......... Química ......... Unicamp/2021 ...................... Múltipla escolha

22 ........... 197596 ..... Elevada ......... Química ......... Unicamp/2021 ...................... Analítica

23 ........... 198483 ..... Elevada ......... Química ......... Ita/2021 ................................ Analítica

Página 102 de 105


Interbits – SuperPro ® Web

24 ........... 194120 ..... Elevada ......... Química ......... Fgv/2020 .............................. Múltipla escolha

25 ........... 189548 ..... Elevada ......... Química ......... Espcex (Aman)/2020 ........... Múltipla escolha

26 ........... 193593 ..... Elevada ......... Química ......... Ita/2020 ................................ Múltipla escolha

27 ........... 182278 ..... Elevada ......... Química ......... Fuvest/2019 ......................... Múltipla escolha

28 ........... 182462 ..... Elevada ......... Química ......... Ita/2019 ................................ Múltipla escolha

29 ........... 182374 ..... Elevada ......... Química ......... Unicamp/2019 ...................... Múltipla escolha

30 ........... 181418 ..... Elevada ......... Química ......... Fgv/2018 .............................. Múltipla escolha

31 ........... 176353 ..... Elevada ......... Química ......... Fuvest/2018 ......................... Analítica

32 ........... 174162 ..... Elevada ......... Química ......... Ime/2018 .............................. Múltipla escolha

33 ........... 176324 ..... Elevada ......... Química ......... Ita/2018 ................................ Múltipla escolha

34 ........... 176345 ..... Elevada ......... Química ......... Ita/2018 ................................ Analítica

35 ........... 178591 ..... Elevada ......... Química ......... Mackenzie/2018 ................... Múltipla escolha

36 ........... 174562 ..... Elevada ......... Química ......... Ime/2018 .............................. Analítica

37 ........... 176326 ..... Elevada ......... Química ......... Ita/2018 ................................ Múltipla escolha

38 ........... 167231 ..... Elevada ......... Química ......... Unicamp/2017 ...................... Analítica

39 ........... 166714 ..... Elevada ......... Química ......... Ita/2017 ................................ Analítica

40 ........... 165827 ..... Elevada ......... Química ......... Unicamp/2017 ...................... Múltipla escolha

41 ........... 168992 ..... Elevada ......... Química ......... Fuvest/2017 ......................... Analítica

42 ........... 166693 ..... Elevada ......... Química ......... Ita/2017 ................................ Múltipla escolha

43 ........... 166705 ..... Elevada ......... Química ......... Ita/2017 ................................ Múltipla escolha

44 ........... 166719 ..... Elevada ......... Química ......... Ita/2017 ................................ Analítica

45 ........... 152589 ..... Elevada ......... Química ......... Mackenzie/2016 ................... Múltipla escolha

46 ........... 153012 ..... Elevada ......... Química ......... Ita/2016 ................................ Múltipla escolha

47 ........... 153954 ..... Elevada ......... Química ......... Unicamp/2016 ...................... Analítica

48 ........... 149484 ..... Elevada ......... Química ......... Ime/2016 .............................. Analítica

49 ........... 153011 ..... Elevada ......... Química ......... Ita/2016 ................................ Múltipla escolha

50 ........... 153020 ..... Elevada ......... Química ......... Ita/2016 ................................ Analítica

51 ........... 153957 ..... Elevada ......... Química ......... Unicamp/2016 ...................... Analítica

52 ........... 152588 ..... Elevada ......... Química ......... Mackenzie/2016 ................... Múltipla escolha

53 ........... 153004 ..... Elevada ......... Química ......... Ita/2016 ................................ Múltipla escolha

Página 103 de 105


Interbits – SuperPro ® Web

54 ........... 137207 ..... Elevada ......... Química ......... Ita/2015 ................................ Múltipla escolha

55 ........... 136361 ..... Elevada ......... Química ......... Unicamp/2015 ...................... Analítica

56 ........... 130930 ..... Elevada ......... Química ......... Upe/2014 ............................. Múltipla escolha

57 ........... 129828 ..... Elevada ......... Química ......... Ita/2014 ................................ Múltipla escolha

58 ........... 124272 ..... Elevada ......... Química ......... Ime/2013 .............................. Múltipla escolha

59 ........... 124282 ..... Elevada ......... Química ......... Ime/2013 .............................. Analítica

60 ........... 124345 ..... Elevada ......... Química ......... Ime/2012 .............................. Múltipla escolha

61 ........... 110402 ..... Elevada ......... Química ......... Fuvest/2012 ......................... Analítica

62 ........... 124347 ..... Elevada ......... Química ......... Ime/2012 .............................. Múltipla escolha

63 ........... 110718 ..... Elevada ......... Química ......... Unicamp/2012 ...................... Analítica

64 ........... 124358 ..... Elevada ......... Química ......... Ime/2012 .............................. Analítica

65 ........... 112297 ..... Elevada ......... Química ......... Upe/2012 ............................. Múltipla escolha

66 ........... 110424 ..... Elevada ......... Química ......... Fuvest/2012 ......................... Analítica

67 ........... 124353 ..... Elevada ......... Química ......... Ime/2012 .............................. Analítica

68 ........... 101914 ..... Elevada ......... Química ......... Fuvest/2011 ......................... Analítica

69 ........... 128204 ..... Elevada ......... Química ......... Ime/2011 .............................. Múltipla escolha

70 ........... 101604 ..... Elevada ......... Química ......... Ita/2011 ................................ Múltipla escolha

71 ........... 128195 ..... Elevada ......... Química ......... Ime/2011 .............................. Múltipla escolha

72 ........... 100945 ..... Elevada ......... Química ......... Fuvest/2011 ......................... Múltipla escolha

73 ........... 101610 ..... Elevada ......... Química ......... Ita/2011 ................................ Múltipla escolha

74 ........... 101617 ..... Elevada ......... Química ......... Ita/2011 ................................ Analítica

75 ........... 106931 ..... Elevada ......... Química ......... Ime/2010 .............................. Múltipla escolha

76 ........... 106939 ..... Elevada ......... Química ......... Ime/2010 .............................. Múltipla escolha

77 ........... 106938 ..... Elevada ......... Química ......... Ime/2010 .............................. Múltipla escolha

78 ........... 106810 ..... Elevada ......... Química ......... Ime/2010 .............................. Analítica

79 ........... 106809 ..... Elevada ......... Química ......... Ime/2010 .............................. Analítica

80 ........... 106808 ..... Elevada ......... Química ......... Ime/2010 .............................. Analítica

81 ........... 28876 ....... Elevada ......... Química ......... Fuvest/1999 ......................... Múltipla escolha

82 ........... 863 ........... Elevada ......... Química ......... Ita/1995 ................................ Múltipla escolha

83 ........... 141265 ..... Elevada ......... Química ......... Ime/2015 .............................. Analítica

Página 104 de 105


Interbits – SuperPro ® Web

Página 105 de 105

Você também pode gostar